Anda di halaman 1dari 40

CHAPTER 20 CHAPTER 20

Carbon and
Hydrocarbons

Chapter Overview
20-1 outlines the covalent
Carbon and
bonding that occurs with
carbon as well as carbon’s
major allotropes.
20-2 describes how to depict
Hydrocarbons
structural formulas and deter-
mine the difference between
structural and geometric isomers.
20-3 describes how to name
saturated hydrocarbons and
explains the relationship
between the structures of al-
kanes and some of their uses.
20-4 describes the naming and
properties of unsaturated
hydrocarbons.

Concept Base
Students may need a review
of the following concepts:
• electron configurations,
Chapter 4
• hybridization and covalent
bonding, Chapter 6
• intermolecular attraction
with liquids and solids,
Chapter 12
• properties of solutes in
solutions, Chapter 13

Reading Skill-Builder
K/W/L Have students list what
they know or want to know about
carbon and the importance of car-
bon compounds. After they have
read the chapter, have them add to
their lists what they have learned.
If they still have questions, have
them research these questions Three-dimensional models help us
using other sources.
visualize the shape of carbon compounds.
624

Copyright © by Holt, Rinehart and Winston. All rights reserved.


SECTION 20-1

SECTION 20-1
Abundance and Lesson Starter
Have students look around the room
and identify all the items that contain

Importance of Carbon OBJECTIVES carbon. Most of the items in view


will in some manner involve carbon.
This might lead to a discussion about
Relate the ability of a carbon our society’s dependence on fossil
atom to form covalent bonds fuels. For example, we depend on
to its atomic structure and fossil fuels for electricity, fuel for
hybrid orbitals. automobiles, plastics, waxes, and
C arbon is found in nature both as an element and in combined form. fibers such as nylon, polyester, and
Although carbon ranks about 17th in abundance by mass among the spandex. Remind students that fossil
Identify the different allotropes
elements in Earth’s crust, it is exceedingly important because it is found fuels are, for all practical purposes,
of carbon and their structural
in all living matter. Carbon is present in body tissues and in the foods nonrenewable resources and are in
differences.
limited supply.
you eat. It is also found in common fuels, such as coal, petroleum, nat-
ural gas, and wood.
Explain how the different
structures of carbon allotropes Visual Strategy
FIGURE 20-1 Be sure students
affect their properties.
understand that a tetrahedron is the
Structure and Bonding of Carbon geometric shape that results when
four corners of a three-dimensional
Carbon, the first member of Group 14, has mostly nonmetallic proper- structure are as far apart as possible.
ties. In its ground state, a carbon atom has an electronic configuration Ask students what the four objects
are (carbon’s valence electrons) and
of 1s22s22p2. The two 1s electrons are tightly bound to the nucleus. The
why they are as far apart as possible
two 2s electrons and the two 2p electrons are the valence electrons. (electron repulsion).
Carbon atoms show a very strong tendency to share electrons and form
Module 4: Chemical Bonding
covalent bonds.
As was covered in Chapter 6, hybridization can be used to explain CHAPTER CONNECTION
the bonding and geometry of most carbon compounds. Carbon atoms
that form four single bonds have four sp3 orbitals. These orbitals are Students should review electron
directed toward the four corners of a regular tetrahedron, as shown in configurations in Chapter 3 and
hybridization in Chapter 6 in order
Figure 20-1. This results in the tetrahedral shape of methane, CH4, and
to better understand this section.
the zigzag pattern of molecules with multiple single-bonded carbon FIGURE 20-1 The orbital models
atoms, such as C4H10. show how the orientation of sp3
hybrid orbitals relates to the
geometry of CH4 and C4H10.
Module 4: Chemical Bonding

H Topics: Covalent/Ionic Tutorial,


Molecular Geometry Tutorials
and Practice
C Sections d, e, f, g, and h of this engag-
ing tutorial review and reinforce
understanding of chemical bonding.
C
CH4
H
H
sp3 hybrid orbitals

CH4 orbital overlap C4H10

CARBON AND HYDROCARBONS 625 625

Copyright © by Holt, Rinehart and Winston. All rights reserved.


SECTION 20-1
(a)

CHAPTER CONNECTION H H

In Chapter 6, students found out how C C C


single bonds are made from hybridized H H
orbitals. Multiple bonds also involve C2H4
some orbitals that are not hybridized.
C4H8
The overlap of unhybridized orbitals
creates the additional bonding in sp2 hybrid orbitals C2H4 orbital overlap
double and triple bonds.
(b)

Visual Strategy C H C C H
FIGURE 20-2 The geometry of
sp hybrid orbitals C2H2
hybridized orbitals is easily demon-
strated with balloons. Obtain one C2H2 orbital overlap
balloon for each hybrid orbital, and
hold the tied ends of the balloons FIGURE 20-2 (a) Three sp2
hybrid orbitals lie in the same plane. C6H10
together. Four balloons will form the
The C2H4 orbital overlap model
shape of a tetrahedron, three bal-
shows the orientation of sp2 hybrid
loons should show a triangular pla-
orbitals in molecules that contain a
nar shape, and two balloons are
double bond, such as C2H4 and C4H8. Carbon atoms form double bonds through sp2 hybridization, as
linear. The balloons, like the electrons (b) The C2H2 orbital overlap model shown in Figure 20-2(a). When carbon atoms form double bonds, the sp2
in hybrid orbitals, will be as far from shows the orientation of sp hybrid hybrid orbitals of both carbon atoms lie in the same plane, as shown in
each other as possible. orbitals in molecules that contain a the orbital overlap model of ethene, C2H4. Because the hydrogen atoms
triple bond, such as C2H2 and C6H10. of C2H4 also bond with carbon sp2 orbitals, all six atoms lie in the same
FIGURE 20-3 Be sure students plane. The three-dimensional models of C2H4 and C4H8 show the geom-
recognize the tetrahedral arrangement etry of molecules containing carbon-carbon double bonds.
of the carbon atoms in a diamond Carbon triple bonds are linear due to the linear arrangement of two
crystal. If the carbon atoms in
sp hybrid orbitals, as shown in Figure 20-2(b). This can be seen in the
diamond show a tetrahedral
orbital overlap model for ethyne, C2H2. The three-dimensional models
geometry, how are the orbitals
in diamond hybridized? of C2H2 and C6H10 show the geometry of molecules containing carbon-
carbon triple bonds.

Allotropes of Carbon
Carbon occurs in several solid allotropic forms that have dramatically
different properties. Diamond is a colorless, crystalline, solid form of
carbon. Graphite is a soft, black, crystalline form of carbon that is a fair
conductor of electricity. Fullerenes are dark-colored solids made of
spherically networked carbon-atom cages.

Diamond
Diamond is the hardest material known. It is the most dense form of
FIGURE 20-3 In diamond, the
carbon atoms are densely packed carbon—about 3.5 times more dense than water. It also has an extreme-
because each carbon atom is bonded ly high melting point (greater than 3500°C). These properties of dia-
to four tetrahedrally oriented carbon mond can be explained by its structure. The model in Figure 20-3 shows
atoms. carbon atoms in diamond bonded covalently in a network fashion. Each

626 626 CHAPTER 20

Copyright © by Holt, Rinehart and Winston. All rights reserved.


SECTION 20-1
carbon atom is tetrahedrally oriented to its four nearest neighbors. The
distance between the carbon-atom nuclei has been measured to be
154 pm. Because of diamond’s extreme hardness and high melting Did You Know?
• In 1772, Antoine Lavoisier heated
point, its major industrial uses are for cutting, drilling, and grinding. NSTA
a diamond with oxygen in a
Diamonds used in industry are not of gem quality. TOPIC: Allotropes sealed container and produced
Another property of diamond is its ability to conduct heat. A dia- GO TO: www.scilinks.org
carbon dioxide. From this, he con-
mond crystal conducts heat more than five times more readily than sil- sci LINKS CODE: HC2201
cluded that diamonds are made
ver or copper, which are the best metallic conductors. In diamond, heat TOPIC: Diamond/graphite entirely from carbon.
is conducted by the transfer of energy of vibration from one carbon GO TO: www.scilinks.org
sci LINKS CODE: HC2202 • The graphite found in pencil
atom to the next. In a diamond crystal, this process is very efficient “lead” is actually mixed with clay.
because the carbon atoms have a small mass. The forces holding the The number rating of the pencil
atoms together are strong and can easily transfer vibratory motion increases as the percentage of
among the atoms. However, unlike metals, diamond does not conduct clay mixed with graphite increases.
electricity. Because all the valence electrons are used in forming local- Even though graphite occurs
ized covalent bonds, none of the electrons can migrate. naturally, most of the graphite
used to make pencils is produced
Graphite in furnaces.
Graphite is nearly as remarkable for its softness as diamond is for its
hardness. It feels greasy and crumbles easily, characteristics that are
CHAPTER CONNECTION
readily explained by its structure. The carbon atoms in graphite are
arranged in layers that form thin hexagonal plates, as shown by the The delocalized electrons described on
model in Figure 20-4. this page form the basis of resonance
The distance between the nuclei of adjacent carbon atoms within a structures, studied in Chapter 6. A
layer has been measured to be 142 pm. This distance is less than the resonance structure is a theoretical
distance between adjacent carbon atom nuclei in diamond. However, combination of two or more struc-
the distance between the nuclei of atoms in adjacent layers measures tures, each showing bonding elec-
335 pm. Because the average distance between carbon atoms in trons in two or more different places.
The electrons in the actual structure
graphite is greater than the average distance in diamond, graphite has
are delocalized over all the atoms
a lower density.
involved. Delocalized electrons
The layers of carbon atoms in graphite are too far apart to be held demonstrate one of the weaknesses
together by covalent bonds. Only weak London dispersion forces hold of using Lewis dot structures to
the layers together. Because of the weak attraction, the layers can slide model covalent bonding: drawing an
across one another. This property allows graphite to be used as a lubri- electron as a single dot assumes that
cant and in pencil “lead.” the electron will remain between two
Within each layer, each carbon atom is bonded to only three other atoms in the dot structure. As in the
carbon atoms. These bonds are examples of resonance hybrid bonds, case of delocalized electrons, this
which were discussed in Chapter 6. The bonding electrons of resonance assumption is not always valid.
hybrid bonds can be thought of as delocalized. Delocalized electrons
are electrons shared by more than two atoms.
HANDBOOK CONNECTION
Graphite is a fairly good conductor of electricity, even though it is a
nonmetal, because the delocalized electrons move freely within each Physical properties of diamond and
layer. Like diamond, graphite has a high melting point (3652°C). This is graphite are found in Group 14 of
because the structure created by delocalized electrons results in a the Elements Handbook.
strongly bonded covalent network. Another use of graphite is in
graphite fibers. Graphite fibers are stronger and stiffer than steel, but
FIGURE 20-4 Notice the space
less dense. The strength of the bonds within a layer makes graphite dif- between layers in the ball-and-stick
ficult to pull apart in the direction parallel to the surface of the layers. model of graphite. Graphite pencils
The strength and light weight of graphite fiber have led to its use in mark on paper because adjacent
products such as sporting goods and aircraft. layers can slide past each other.

CARBON AND HYDROCARBONS 627 627

Copyright © by Holt, Rinehart and Winston. All rights reserved.


SECTION 20-1

Did You Know?


• The structure of buckyballs makes
them very stable—even harder
than diamonds at high pressures.
• When one of the carbon atoms in
a buckyball is replaced by silicon,
the buckyball becomes a semicon-
ductor. When a potassium atom is
placed in the center of a bucky-
ball, it becomes a superconductor
(b)
at low temperatures. These and
other traits make buckyballs
attractive in the development of
new plastics and medicines. Have
students determine the hybridiza- (a) (c)
tion of carbon atoms in buckmin-
FIGURE 20-5 (a) Buckminster-
sterfullerene.
fullerene was named after
Buckminster Fuller, who invented Fullerenes
geodesic domes like the one shown In the mid-1980s a new allotropic form of carbon was discovered. The
SECTION REVIEW here. (b) The structure of buckmin- 1996 Nobel Prize in chemistry was awarded to Richard E. Smalley,
1. Carbon is present in all living sterfullerene resembles the pattern Robert F. Curl, and Harold W. Kroto, leaders of the research teams that
matter. of a soccer ball (c).
discovered this class of compounds, fullerenes.
2. sp 2, sp Fullerenes are part of the soot that forms when carbon-containing
3. The carbon atoms in graphite are materials are burned with limited oxygen. Their structures consist of
arranged in layers of thin, hexagonal near-spherical cages of carbon atoms. The most stable of these is C60,
plates that are strong in the direction shown in Figure 20-5. C60 is formed by 60 carbon atoms arranged in
NSTA
of the layers but slide across one interconnected five- and six-membered rings.
another. The strength of graphite in TOPIC: Buckminster Fuller Because of its structural resemblance to geodesic domes, Richard
GO TO: www.scilinks.org
the direction of the layers explains Smalley and his co-workers at Rice University named C60 “buckmin-
sci LINKS CODE: HC2203
the strength of carbon fibers. The sterfullerene” in honor of the geodesic-dome architect, Buckminster
sliding of the layers explains the Fuller. The whole family of carbon-atom cages, which have a wide range
softness of graphite and why it is in the number of carbon atoms, are therefore called fullerenes. Because
used as a lubricant. the structure of C60 also resembles the design of a soccer ball, C60 is also
4. a. All fullerenes have a near- known less formally as buckyball. Scientists are currently trying to find
spherical cagelike structure. practical uses for these substances.
b. Fullerenes vary in the number of
carbon atoms in the molecule.

SECTION REVIEW
1. What makes carbon an important element in the 3. How does the structure of graphite relate to its
study of chemistry? properties and uses?
2. What type of hybrid orbital is found in carbon 4. a. How are the structures of different fullerenes
double bonds? In carbon triple bonds? similar?
b. How do they differ?

628 628 CHAPTER 20

Copyright © by Holt, Rinehart and Winston. All rights reserved.


SECTION 20-2

SECTION 20-2
Organic Compounds Lesson Starter
Begin this lesson with a group-puzzle
exercise. Before class, draw twenty
1.5 × 1.5 in. boxes on a sheet of
OBJECTIVES paper. Draw a circle in each box.
Write the letter C inside six of the
Explain how the structure and circles, and add four lines from the
A ll organic compounds contain carbon atoms. However, not all bonding of carbon lead to the circle to the sides of the square for
carbon-containing compounds are classified as organic. There are a few diversity and number of four bonds. Write the letter H inside
exceptions, such as Na2CO3, CO, and CO2, that are considered inor- organic compounds. the remaining 14 circles, and add a
ganic. Organic compounds, then, can be defined as covalently bonded single line from the circle to one side
compounds containing carbon, excluding carbonates and oxides. Figure of the square for one bond.
Explain the importance and
20-6 shows a few familiar items that contain organic compounds. Divide students into groups of
limitations of molecular and
three. Provide each group with a copy
structural formulas.
of the paper and a pair of scissors.
Ask them to develop as many differ-
Carbon Bonding and the Diversity Compare structural and ent molecules as possible in five
geometric isomers. minutes using all of the atoms and all
of Organic Compounds of the bonds. Compare the findings.
Ask the students how they could do
The diversity of organic compounds results from the uniqueness of car- this exercise three-dimensionally.
bon’s structure and bonding. Carbon’s electronic structure allows it to Would anything change? How would
bind to itself to form chains and rings, to bind covalently to other ele- it be different?
ments, and to bind to itself and other elements in different arrangements.

✔Teaching Tip
FIGURE 20-6 Aspirin, polyethyl-
Using models will help students
ene in plastic bags, citric acid in fruit,
visualize the structures of the mol-
and amino acids in animals are all
ecules covered in this chapter. Many
examples of organic compounds.
commercial ball-and-stick and
space-filling models are available.
It is a good idea to have at least one
of these for demonstration purposes.
Models can also be made using gum-
drops as atoms and toothpicks as
bonds. Different colors and sizes of
gumdrops can be used for different
atoms. Start by having the students
insert four toothpicks into one gum-
drop so that they point to the four
corners of a tetrahedron. This model,
representing a carbon atom with sp3
hybridized orbitals, can be used as a
pattern to form chains and rings.
Save these models to use throughout
the chapter.
Safety: Discourage all eating in
the lab because of possible contami-
nation.

CARBON AND HYDROCARBONS 629 629

Copyright © by Holt, Rinehart and Winston. All rights reserved.


SECTION 20-2
Carbon-Carbon Bonding


Carbon atoms are unique in their ability to form long chains and rings
Teaching Tip of covalently bonded atoms. This type of bonding is known as catena-
• The structural formula of ethanol: tion, the covalent binding of an element to itself to form chains or rings.
H H This produces a multitude of chain, branched-chain, and ring structures.
In addition, carbon atoms in these structures can be linked by single,
HICICIOH
double, or triple covalent bonds. Examples of molecules containing car-
H H bon-atom rings and chains are shown in Figure 20-7.
• The structural formula of dimethyl
ether: Carbon Bonding to Other Elements
H H Besides binding to other carbon atoms, carbon atoms bind readily to
HICIOICIH elements with similar electronegativities. Organic compounds consist of
carbon and these other elements. Hydrocarbons are composed of only
H H
carbon and hydrogen; they are the simplest organic compounds. Other
• Be sure students recognize the organic compounds contain hydrocarbon backbones to which other ele-
difference between a two- ments, primarily O, N, S, and the halogens, are attached. Figure 20-8
dimensional representation and a
shows a molecule in which carbon atoms are bound to other elements.
three-dimensional representation.
A two-dimensional representation
Arrangement of Atoms
of CH4 appears to have 90o angles
between hydrocarbon bonds. A The bonding capabilities of carbon also allow for different arrange-
FIGURE 20-7 Compare the shape
three-dimensional representation of a fatty acid found in cream with
ments of atoms. This means that some compounds may contain the
shows the more realistic 109o that of fructose, found in fruit. In the same atoms but have different properties because the atoms are
angle. As students write and use fatty acid, the carbon atoms are in arranged differently. For example, the molecular formula C2H6O repre-
structural formulas, constantly chains. In fructose, carbon atoms sents both ethanol and dimethyl ether. Compounds that have the same
remind them that as they write a form a ring. molecular formula but different structures are called isomers. As the
two-dimensional representation, number of carbon atoms in a molecular formula increases, the number
they should be thinking in three of possible isomers increases rapidly. For example, there are 18 isomers
dimensions. with the molecular formula C8H18, 35 with the molecular formula
C9H20, and 75 with the molecular formula C10H22. For the molecular
Reading Skill-Builder formula of just 40 carbon atoms and 82 hydrogen atoms, C40H82, there
INTERPRETING are theoretically 69 491 178 805 831 isomers. To distinguish one from
VOCABULARY Explain that the another, more information than just the molecular formula is needed.
word catenation comes from the
Latin word catena, which means
“chain.” Tell students that carbon Structural Formulas
is one of just a few elements capa-
ble of forming chains and rings by For this reason, organic chemists use structural formulas to represent
bonding to other carbon atoms. organic compounds. A structural formula indicates the number and types
Silicon, for example, also exhibits of atoms present in a molecule and also shows the bonding arrangement
some catenation, but carbon of the atoms. For example, one possible structural formula for an isomer
exhibits catenation more than any
of C4H10 is the following.
other element does.
H H H
HIC C CIH
H HICIH H
FIGURE 20-8 In firefly luciferin, H
carbon atoms bind to hydrogen,
oxygen, nitrogen, and sulfur. Structural formulas are sometimes condensed to make them easier
Luciferin is responsible for the light to read. In one type of condensed structure, hydrogen single covalent
emitted from the tail of a firefly. bonds are not shown. The hydrogen atoms are understood to bind to the

630 630 CHAPTER 20

Copyright © by Holt, Rinehart and Winston. All rights reserved.


SECTION 20-2
FIGURE 20-9 The structure
H OH of ethanol can be represented in
different ways. Ball-and-stick and
Visual Strategy
HICIICIH FIGURE 20-9 Have students make
space-filling models represent the
H H three-dimensional shape of the
a ball-and-stick model for ethanol
molecule. using gumdrops and toothpicks. Use
large gumdrops of one color for C
Ball-and-stick model Space-filling model and another color for O. Use small
gumdrops for H. After the students
have finished the model, have them
atom they are written beside. The following structural and condensed push the gumdrops together into a
structural formulas represent the same molecule. compact form to approximate the
space-filling model.
H H H
CH3ICHICH3
✔Teaching Tip
HIC C CIH
is the same as
H HICIH H CH3
H Some students may have difficulty
recognizing whether two molecules
Remember that the structural formula does not accurately show the are isomers or actually the same
three-dimensional shape of the molecule. Three-dimensional shape is molecule from a different perspec-
depicted with drawings or models, as shown for ethanol in Figure 20-9. tive. This visual reasoning is quite
As you continue your study, you may find that the use of dashes can different from the reasoning required
be eliminated by writing in a horizontal row the symbols and subscripts to solve problems. Look for students
for the groups of carbon and hydrogen atoms that appear in a molecule. who can easily recognize and deter-
mine whether formulas represent
For example, ethane is written as CH3CH3 and propane as CH3CH2CH3.
isomers. This may be an opportunity
to acknowledge a student who does
not do well in the mathematics-
heavy portions of the course.
Isomers
You have learned that isomers are compounds that have the same mol-
ecular formula but different structural formulas. Isomers can be further
DEMONSTRATION
Have students use models to con-
classified by structure and geometry.
struct the isomers of C5H12 (pentane)
and C6H14 (hexane). Have them draw
Structural Isomers the structural formulas for each iso-
Structural isomers are isomers in which the atoms are bonded together in mer they build. How does the num-
different orders. For example, the atoms of the molecular formula C4H10 ber of possible isomers of pentane
can be arranged in two different ways. compare with the number of possible
hexane isomers? (There are many
H H H more hexane isomers than pentane
H H H H
HIC C CIH isomers.)
HICICICICIH
H HICIH H
H H H H
H
butane methylpropane ✔Teaching Tip
In modern usage, structural isomers
Notice that the formula for butane shows a continuous chain of four
are also known as constitutional
carbon atoms. The chain may be bent or twisted, but it is continuous.
isomers.
The formula of methylpropane shows a continuous chain of three car-
bon atoms, with the fourth carbon atom attached to the second carbon
atom of the chain.

CARBON AND HYDROCARBONS 631 631

Copyright © by Holt, Rinehart and Winston. All rights reserved.


SECTION 20-2
TABLE 20-1 Physical Properties of the Structural
Isomers Butane and Methylpropane
TABLE STRATEGY
Melting point Boiling point Density at 20°C
Table 20-1 Point out the differences (°C) (°C) (g/mL)
in melting and boiling points for the
two butane isomers, and then explain butane −138.4 −0.5 0.5788
the difference in terms of intermolecu- methylpropane −159.4 −11.633 0.549
lar attraction.

Common Structural isomers can have different physical or chemical proper-


Misconception ties. For example, butane and methylpropane have different melting
It is easy for students to forget that
points, boiling points, and densities, as shown in Table 20-1.
there is free rotation around single
bonds but not around double bonds.
The different structural formulas Geometric Isomers
available for a molecule as a result Geometric isomers are isomers in which the order of atom bonding is
of bond rotation may appear to some the same but the arrangement of atoms in space is different. Consider
students as different isomers. Draw the molecule 1,2-dichloroethene, which contains a double bond. The
2,3-dimethylbutane on the board double bond prevents free rotation and holds groups to either side of
with three different bond rotations. the molecule. This means there can be two different 1,2-dichloroethene
For example: geometric isomers as shown below.
H H
Cl Cl H Cl
CIIIC
CH3 CH3 CJC CJC
CH3 CH3 H H Cl H
Point out why the three different cis trans
representations are really the same
molecule. Demonstrate this with Because the two chlorine atoms are on the same side of the molecule
models. in the first structure, it is called cis. In the second molecule, the two chlo-
rine atoms are on opposite sides of the molecule, and so the molecule is
called trans. Notice that in both molecules the bonding order of the
atoms is the same: each carbon atom in the double bond is also bound
to one chlorine atom and one hydrogen atom.
Now consider the molecule 1,2-dichloroethane. Atoms attached to
the carbon atoms can rotate freely around the single carbon-carbon
bond, as shown in Figure 20-10. There are no geometric isomers of 1,2-
dichloroethane. In order for geometric isomers to exist, there must be a
rigid structure in the molecule to prevent free rotation around a bond.
Now consider two apparent structures for another molecule with a
double bond, chloroethene.
H Cl H H
CJC CJC
H H H Cl
FIGURE 20-10 Unlike double
Although these structures may appear different at first glance, they
bonds, single bonds allow free rota-
tion within a molecule. Groups
are actually the same. In both structures, two hydrogen atoms are on
attached to the carbon atoms are not one side of the molecule, and one chlorine atom and one hydrogen atom
held to one side of the molecule, so are on the other. A molecule can have a geometric isomer only if two car-
there are no geometric isomers. bon atoms in a rigid structure each have two different groups attached.

632 632 CHAPTER 20

Copyright © by Holt, Rinehart and Winston. All rights reserved.


SECTION 20-2
O FIGURE 20-11 Males of the Iowa

✔Teaching Tip
CH3CH2 (CH2)9CH2OCCH3 strain of the European corn borer
CJC respond most strongly to mixtures of
H H the female sex attractant pheromone
Students can determine whether
that are 96% cis isomer. But males
cis-11-tetradecenyl acetate of the New York strain respond most
two molecules are geometric isomers
strongly to mixtures containing 97% by trying to superimpose three-
O
trans isomer. dimensional models on top of one
H (CH2)9CH2OCCH3 another. Simplify the models by rep-
CJC resenting each group attached to a
CH3CH2 H carbon atom in the double bond with
trans-11-tetradecenyl acetate a single ball of a different color. If
students cannot turn or twist one
molecule so that it matches the other
exactly, then the two molecules are
isomers.
Like structural isomers, geometric isomers differ in physical and chemi-
cal properties. Some geometric isomers are known to differ in physiologi-
cal behavior as well. For example, insects can communicate by chemicals
called pheromones and may distinguish between the geometric isomers
of pheromones. One geometric isomer of a pheromone may be physio-
logically active, while the other will be only slightly active or not at all.
The European corn borer, shown in Figure 20-11, distinguishes between
isomers of its sex-attractant pheromone. Another example of differences
between geometric isomers is found in fatty acids. Natural unsaturated SECTION REVIEW
fatty acids are primarily cis-fatty acids. Hydrogenation is used to convert
vegetable oil, which contains unsaturated fatty acids, into a solid fat, such 1. its ability to bind to itself to form
chains and rings, its ability to bind
as margarine or vegetable shortening. During hydrogenation trans-fatty
with other elements, and its ability
acids are produced. Research has shown that there may be health risks
to bind to itself and other elements
associated with diets high in trans-fatty acids. in different arrangements
2. Compounds that have the same
molecular formula but different
structures are isomers. Structural
SECTION REVIEW isomers have atoms bonded together
in a different order. Geometric isomers
1. What are three characteristics of carbon that con- 5. Which of the following can represent the same have atoms bonded in the same
tribute to the diversity of organic compounds? molecule? order, but the arrangement of atoms
in space is different.
2. Define the term isomer, and distinguish between a. H H H H H
structural and geometric isomers. 3. a. Molecular formulas cannot
HICICICICICIH show the difference between isomers
3. Which of the following types of molecular represen- because they show only the ratio of
H H H H H
tations can be used to show differences between atoms involved. They do not show
isomers? Explain why each can or cannot. b. CH3ICH2ICH2ICH3 how the atoms are arranged.
a. molecular formula b. and c. A structural formula and
b. structural formula c. CH3ICH2ICH2 a three-dimensional drawing, or
c. three-dimensional drawing or model model, can be used to show differ-
CH2
4. Write the formula for methylpropane (shown at the ences between isomers because they
right on page 631) in a horizontal row. CH3 can show the order atoms are bonded
together and arranged in space.
d. C5H12
4. CH3CHCH3CH3
5. a., c., and d.

CARBON AND HYDROCARBONS 633 633

Copyright © by Holt, Rinehart and Winston. All rights reserved.


SECTION 20-3

SECTION 20-3
Lesson Starter
Write the structural formulas of
methane, ethane, propane, and
Saturated
butane on the board. Ask students
to predict the formula for the next
hydrocarbon in the series. Use these
OBJECTIVES Hydrocarbons
structures to introduce the alkane Recognize the important
homologous series. Have students structural feature of satu-
confirm that each formula follows rated hydrocarbons, alkanes.
the Cn H2n +2 rule for saturated
alkanes. Have students predict the
Be able to name and write H ydrocarbons are grouped mainly by the type of bonding between
molecular formula for the alkane carbon atoms. Saturated hydrocarbons are hydrocarbons in which each
structural formulas for
with 15 carbon atoms. carbon atom in the molecule forms four single covalent bonds with other
alkanes.
atoms.
Reading Skill-Builder
Explain how structures
READING ORGANIZER Have of alkanes relate to their
students read Section 20-3 and properties and how those
organize the information in a table properties affect the uses Alkanes
titled “Organic Compounds.” of specific alkanes.
Students should include informa- Hydrocarbons that contain only single bonds are alkanes. In Table 20-2,
tion on basic structural characteris- the molecular formulas, structural formulas, and space-filling models
tics, rules for naming, properties, are given for alkanes with one to four carbon atoms. If you examine the
uses, and examples of saturated molecular formulas for successive alkanes in Table 20-2, you will see a
hydrocarbons. Students should also
clear pattern. Each member of the series differs from the preceding one
list any subcategories that exist,
by one carbon atom and two hydrogen atoms. For example, propane,
such as alkanes and cycloalkanes.
Finally, students should draw a NSTA
C3H8, differs from ethane, C2H6, by one carbon atom and two hydrogen
condensed structural formula for atoms, a ICH2I group.
TOPIC: Alkanes
representative examples and cor- GO TO: www.scilinks.org
sci LINKS CODE: HC2204 H H H H H
rectly name them. Tell students to
keep their tables so they can add HICICIH HICICICIH
to them as they learn more about H H H H H
organic compounds.
ethane propane

Compounds that differ in this fashion belong to a homologous series.


A homologous series is one in which adjacent members differ by a con-
stant unit. It is not necessary to remember the molecular formulas for all
members of a homologous series. Instead, a general molecular formula
can be used to determine the formulas. Look at the molecular formulas
for ethane and propane, C2H6 and C3H8. They both fit the formula
CnH2n+2. For ethane, n = 2, so there are two carbon atoms and (2 × 2) +
2 = 6 hydrogen atoms. For propane, n = 3, so there are three carbon
atoms and (2 × 3) + 2 = 8 hydrogen atoms. Now consider a molecule for
which we do not know the molecular formula. Suppose a member of
this series has 30 carbon atoms in its molecules. Then n = 30, and there
are (2 × 30) + 2 = 62 hydrogen atoms. The formula is C30H62.
Notice that for alkanes with three or fewer carbon atoms, only one
molecular structure is possible. However, in alkanes with more than
three carbon atoms, the chains can be straight or branched. Thus,

634 634 CHAPTER 20

Copyright © by Holt, Rinehart and Winston. All rights reserved.


SECTION 20-3
TABLE 20-2 Alkanes with One to Four Carbon Atoms
Molecular formulas Structural formulas Space-filling models
TABLE STRATEGY
H Table 20-2 Point out that each of
HIC IH the formulas is part of a homologous
CH4 series—they differ by a constant
H unit. Ask students why there are no
methane isomers listed for the hydrocarbons
with only one, two, or three carbon
atoms.
H H
HIC IC IH
C2H6
H H
ethane

H H H
HICIC ICIH
C3H8
H H H

propane

H H H H
HICI CI C IC IH
H H H H
butane
C4H10
H H H
HIC C CIH
H HICIH H
H
methylpropane

alkanes with four or more carbon atoms have structural isomers. There
are two possible structural isomers for alkanes with four carbon atoms,
butane and methylpropane.

Cycloalkanes
Cycloalkanes are alkanes in which the carbon atoms are arranged in a
ring, or cyclic, structure. The structural formulas for cycloalkanes are
often drawn in a simplified form. It is understood that there is a carbon

CARBON AND HYDROCARBONS 635 635

Copyright © by Holt, Rinehart and Winston. All rights reserved.


SECTION 20-3
atom at each corner and enough hydrogen atoms to complete the four

✔Teaching Tip
bonds to each carbon atom.

• If hydrocarbon molecules do not CH2


follow the Cn H2n +2 rule, then they CH2 CH2 or
must either be cyclic hydrocarbons CH2 CH2
or have multiple bonds. Hydro-
cyclopentane cyclopentane
carbons with double and triple
bonds are called unsaturated Because there are no free ends where a carbon atom is attached to
because the carbon atoms are not three hydrogen atoms, there are two fewer hydrogen atoms in
saturated with hydrogen to the cycloalkanes than in noncyclic alkanes.
greatest extent possible.
• The IUPAC nomenclature used in H H
this text is an international stan- H H H H
HICICIH
dard. However, older common HICICICICIH
names, such as ethylene for HICICIH
H H H H
ethene, propylene for propene, H H
acetylene for ethyne, and ethyl butane cyclobutane
alcohol for ethanol (in Chapter 21),
C4H10 C4H8
are still in use. These common
names can also be found in The
Merck Index. The general structure for cycloalkanes, CnH2n , shows that they have
2 × n hydrogen atoms. This is two fewer hydrogen atoms than noncyclic
alkanes, CnH2n+2, which have (2 × n) + 2 hydrogen atoms.
TABLE STRATEGY
Table 20-3 Have students think of
common terms that use the same
prefixes as those in this table. Some Systematic Names of Alkanes
examples include pentagon, octopus,
and decade. Historically, the names of many organic compounds were derived from
the sources in which they were found. As more organic compounds
were discovered, a systematic naming method became necessary. The
TABLE 20-3 Carbon-Atom systematic method used primarily in this book was developed by the
Chain Prefixes International Union of Pure and Applied Chemistry, IUPAC.
The basic part of the systematic name of an organic compound is the
Number of
carbon atoms Prefix name of the longest carbon chain, or parent hydrocarbon, in the mol-
ecule. Table 20-3 gives the names of the prefixes for carbon-atom chains
1 meth-
up to 10 carbon atoms long. Beginning with pent-, the prefixes are
2 eth- Greek or Latin numerical prefixes.
3 prop-
4 but- Unbranched-Chain Alkane Nomenclature
To name an unbranched alkane, find the prefix in Table 20-3 that corre-
5 pent-
sponds to the number of carbon atoms in the chain of the hydrocarbon.
6 hex- Then add the suffix -ane to the prefix. An example is shown below.
7 hept-
1 2 3 4 5 6 7
8 oct- CH3ICH2ICH2ICH2ICH2ICH2ICH3
9 non- heptane
10 dec- The molecule has a chain seven carbon atoms long, so the prefix hept-
is added to the suffix -ane to form heptane.

636 636 CHAPTER 20

Copyright © by Holt, Rinehart and Winston. All rights reserved.


SECTION 20-3
TABLE 20-4 Some Straight-Chain Alkyl Groups
Alkane Name Alkyl group Name TABLE STRATEGY
CH4 methane ICH3 methyl Table 20-4 Write the structures
CH3ICH3 ethane ICH2ICH3 ethyl of 3-ethyl-2,4,5-trimethyloctane and
CH3ICH2ICH3 propane ICH2ICH2ICH3 propyl 5-butyl-2,2-dimethylnonane on the
board. Have students identify all of
CH3ICH2ICH2ICH3 butane ICH2ICH2ICH2ICH3 butyl the alkyl groups.
CH3ICH2ICH2ICH2ICH3 pentane ICH2ICH2ICH2ICH2ICH3 pentyl

✔Teaching Tip
Point out that when drawing a
Branched-Chain Alkane Nomenclature structural formula, it is important
The naming of branched-chain alkanes also follows a systematic to write the longest hydrocarbon
method. The hydrocarbon branches of alkanes are alkyl groups. Alkyl sequence first. When given a
groups are groups of atoms that are formed when one hydrogen atom is structural formula, students should
removed from an alkane molecule. Alkyl groups are named by replacing remember that the longest chain may
the suffix -ane of the parent alkane with the suffix -yl. Some examples not be represented in a straight line.
are shown in Table 20-4. Alkyl group names are used when naming
branched-chain alkanes. We will only present the method for naming
simple branched-chain alkanes with only straight-chain alkyl groups.
Consider the following molecule.

CH3 CH3
CH3ICH2ICH2ICHICHICHICH2ICH3
CHICH3
CH3

To name this molecule, locate the parent hydrocarbon. The parent


hydrocarbon is the longest continuous chain that contains the most
straight-chain branches. In this molecule, there are two chains that are
eight carbon atoms long. The parent hydrocarbon is the chain that con-
tains the most straight-chain branches. Do not be tricked by the way the
molecule is drawn. The longest chain may be shown bent.

CH3 CH3
CH3ICH2ICH2ICHICHICHICH2ICH3
CHICH3
CH3

NOT

CH3 CH3
CH3ICH2ICH2ICHICHICHICH2ICH3
CHICH3
CH3

CARBON AND HYDROCARBONS 637 637

Copyright © by Holt, Rinehart and Winston. All rights reserved.


SECTION 20-3
To name the parent hydrocarbon, add the suffix -ane to the prefix

✔Teaching Tip
oct- (for a carbon-atom chain with eight carbon atoms) to form octane.
Now identify and name the alkyl groups.
Have students notice that all alkanes CH3 CH3
end with the suffix -ane. This signi-
CH3ICH2ICH2ICHICHICHICH2ICH3
fies that there are no double bonds.
Also, the resulting numbers in the CHICH3
name should be the lowest possible CH3
numbers.
The three ICH3 groups are methyl groups. The ICH2ICH3 group is an
ethyl group. Arrange the names in alphabetical order in front of the
name of the parent hydrocarbon.

ethyl methyloctane

To show that there are three methyl groups present, attach the prefix
tri- to the name methyl to form trimethyl.

ethyl trimethyloctane

Now we need to show the locations of the alkyl groups on the parent
hydrocarbon. Number the octane chain so that the alkyl groups have
the lowest numbers possible.

CH3 CH3
8 7 6 5 4 3
CH3ICH2ICH2ICHICHICHICH2ICH3
2 CHICH3

1CH3

NOT

CH3 CH3
1 2 3 4 5 6
CH3ICH2ICH2ICHICH2ICHICH2ICH3
7CHICH3

8 CH3

Place the location numbers of each of the alkyl groups in front of its
name. Separate the numbers from the names of the alkyl groups with
hyphens. The ethyl group is on carbon 3.
3-ethyl trimethyloctane
Because there are three methyl groups, there will be three numbers,
separated by commas, in front of trimethyl.
3-ethyl-2,4,5-trimethyloctane
The full name is 3-ethyl-2,4,5-trimethyloctane.
The procedure for naming simple branched-chain alkanes can be sum-
marized as follows.

638 638 CHAPTER 20

Copyright © by Holt, Rinehart and Winston. All rights reserved.


SECTION 20-3
Alkane Nomenclature
1. Name the parent hydrocarbon. Find the longest continuous chain ADDITIONAL
of carbon atoms with straight-chain branches. Add the suffix -ane
SAMPLE
to the prefix corresponding to the number of carbon atoms in the
PROBLEM
chain.
2. Add the names of the alkyl groups. Add the names of the alkyl Additional Sample Problem is found
groups in front of the name of the parent hydrocarbon in alpha- on page 661A.
betical order. When there is more than one branch of the same
alkyl group present, attach the appropriate numerical prefix to the
name, di = 2, tri = 3, tetra = 4, and so on. Do this after the names
have been put in alphabetical order.
3. Number the carbon atoms in the parent hydrocarbon. If one or
more alkyl groups are present, number the carbon atoms in the
continuous chain to give the lowest numbers possible in the name.
If there are two equivalent lowest positions with two different
alkyl groups, give the lowest number to the alkyl group that comes
first in the name. (This will be the alkyl group that is first in alpha-
betical order, before any prefixes are attached.)
4. Insert position numbers. Put the position numbers of each alkyl
group in front of the name of that alkyl group.
5. Punctuate the name. Separate the position numbers from the
names with hyphens. If there are more than one number in front
of a name, separate the numbers by commas.

SAMPLE PROBLEM 20- 1


Name the following simple branched-chain alkane:
CH3ICHICH2ICHICHICH3
CH3 CH3 CH3

SOLUTION 1. Identify and name the parent hydrocarbon.


CH3ICHICH2ICHICHICH3
CH3 CH3 CH3
Because the longest continuous chain contains six carbon atoms, the parent hydrocarbon
is hexane.

2. Identify and name the alkyl groups attached to the chain.


CH3ICHICH2ICHICHICH3
CH3 CH3 CH3
There is only one type of alkyl group, with one carbon atom. Alkyl groups with one car-
bon atom are methyl groups. Add the name methyl in front of the name of the continu-
ous chain. Add the prefix tri- to show that there are three methyl groups present.
trimethylhexane

CARBON AND HYDROCARBONS 639 639

Copyright © by Holt, Rinehart and Winston. All rights reserved.


SECTION 20-3
3. Number the carbon atoms in the continuous chain so that the alkyl groups have the
lowest numbers possible.
ADDITIONAL
6 5 4 3 2 1
SAMPLE CH3ICHICH2ICHICHICH3
PROBLEM
CH3 CH3 CH3
20-2 Draw the condensed structural
formula of each of the following
4. The methyl groups are on the carbon atoms numbered 2, 3, and 5. Put the numbers of
molecules:
the positions of the alkyl groups, separated by commas, in front of the name of the alkyl
a. 2,4-dimethylpentane group. Separate the numbers from the name with a hyphen.
b. 4-ethyl-3-methylheptane
2,3,5-trimethylhexane
c. 2-methylpropane
See page 661A for answers.
The complete name is 2,3,5-trimethylhexane.

SAMPLE PROBLEM 20- 2


Draw the condensed structural formula of 3-ethyl-4-methylhexane.

SOLUTION 1. Identify the name of the parent hydrocarbon.


3-ethyl-4-methylhexane
The parent hydrocarbon is hexane, so there are six carbon atoms in the chain. Draw and
number the carbon atoms in the chain.
1 2 3 4 5 6
C C C C C C

2. Identify the alkyl groups, and determine the number of carbon atoms in the alkyl groups.
3-ethyl-4-methylhexane
Methyl groups have one carbon atom and ethyl groups have two carbon atoms.
H H H
ICIH ICICIH
H H H

3. Locate the position numbers for the ethyl and methyl groups.
3-ethyl-4-methylhexane
Draw the alkyl groups on the parent hydrocarbon in the correct positions.
H
HICIH
1 2 3 4 5 6
C C C C C C
HICIH
HICIH
H

640 640 CHAPTER 20

Copyright © by Holt, Rinehart and Winston. All rights reserved.


SECTION 20-3
Notice that in this molecule the methyl group and the ethyl group were in equivalent
positions from the end of the chain. They are both on third carbons from the end. In
such a case, the alkyl group that comes first in the name is given the lower number. ADDITIONAL
SAMPLE
PROBLEMS
4. Add the correct number of hydrogen atoms so that each carbon atom has four single
bonds. This is the complete, uncondensed, structural formula. 20-2 Draw the condensed structural
formula for 3,3,4-triethyl-4-methyl-
H
hexane.
H H H HICIH H H See page 661A for answer.
HIC C C C C CIH 20-2 Draw the structural formulas
H H HICIH H H H for two structural isomers of C7H16.
See page 661A for answer.
HICIH
20-2 Draw the condensed structural
H
formula for 3-ethyl-2,2-dimethyl-
pentane.
5. To draw the condensed structural formula, show only the bonds between carbon atoms. See page 661A for answer.

CH3
CH3ICH2ICHICHICH2ICH3
CH2
CH3

PRACTICE 1. Name the following molecule: Answer


methylbutane
CH3ICHICH2ICH3
CH3

2. Draw the condensed structural Answer


formula for 3,3-diethyl-2,5-
CH3
dimethylnonane.
CH2
CH3ICHICICH2ICHICH2ICH2ICH2ICH3
CH3 CH2 CH3
CH3

3. Draw the condensed structural Answer


formulas for the two structural
CH3ICHICH2ICH2ICH3
isomers of methylpentane and
name the isomers. CH3
2-methylpentane

CH3ICH2ICHICH2ICH3
CH3
3-methylpentane

CARBON AND HYDROCARBONS 641 641

Copyright © by Holt, Rinehart and Winston. All rights reserved.


SECTION 20-3
Cycloalkane Nomenclature
When naming simple cycloalkanes, the cycloalkane is the parent hydro-
Additional Example carbon. Cycloalkanes are named by adding the prefix cyclo- to the name
Problems of the straight-chain alkane with the same number of hydrocarbons.
1. Name the following cycloalkanes:
a.
CH2
CH3ICH2ICH3
b.
CH2 CH2
CH3ICH2I ICH2ICH3
propane cyclopropane
Ans. a. cyclobutane
b. 1,4-diethylcyclohexane When there is only one alkyl group attached to the ring, no position
2. Draw the condensed structural number is necessary. When there is more than one alkyl group attached
formula for methylcyclopentane. to the ring, the carbon atoms in the ring are numbered to give the low-
est numbers possible to the alkyl groups. This means that one of the
Ans. CH3 alkyl groups will always be in position 1.

CH3 CH3
1 1
6 2 NOT 2 6
5 3 3 5
4 4
CH3 CH3
1,3-dimethylcyclohexane 1,5-dimethylcyclohexane

The rules for naming cycloalkanes can be summarized as follows.

Cycloalkane Nomenclature
Use the rules for alkane nomenclature on page 639, with the
following exceptions.
1. Name the parent hydrocarbon. Count the number of carbon
atoms in the ring. Add the prefix cyclo- to the name of the corre-
sponding straight-chain alkane.
2. Add the names of the alkyl groups.
3. Number the carbon atoms in the parent hydrocarbon. If there are
two or more alkyl groups attached to the ring, number the carbon
atoms in the ring. Assign position number one to the alkyl group
that comes first in alphabetical order. Then, number in the direc-
tion that gives the next lowest number.
4. Insert position numbers.
5. Punctuate the name.

Two examples of correctly named cycloalkanes are given below.

CH3
CH3 CH3

methylcyclohexane 1,1-dimethylcyclobutane

642 642 CHAPTER 20

Copyright © by Holt, Rinehart and Winston. All rights reserved.


SECTION 20-3
TABLE 20-5 Properties of Straight-Chain Alkanes
Molecular State at room TABLE STRATEGY
formula IUPAC name Boiling point (°C) temperature
Table 20-5 Be sure students
CH4 methane −164 gas recognize that the boiling point of
C2H6 ethane −88.6 the alkane increases as the number
C3H8 propane −42.1 of carbon atoms increases. This is
C4H10 butane −0.5 also reflected in the state of the
C5H12 pentane 36.1 liquid alkane at room temperature. Refer
C8H18 octane 125.7 students back to the demonstration
C10H22 decane 174.1 involving Velcro. The longer the strip
C17H36 heptadecane 301.8 solid of Velcro, the more difficult it was
C20H42 eicosane 343 to separate it from another strip.
Similarly, the longer the straight-
chain alkane is, the greater its
attraction to other alkanes and the
higher its boiling point.

Properties and Uses of Alkanes


CHAPTER CONNECTION
Properties for some straight-chain alkanes are listed in Table 20-5. The
trends in these properties can be explained by examining the structure Students may need to review inter-
molecular attraction in Chapter 6.
of alkanes. The carbon-hydrogen bonds of alkanes are nonpolar. The
only forces of attraction between nonpolar molecules are weak inter-
molecular forces, or London dispersion forces. The strength of London
dispersion forces increases as the mass of a molecule increases.

Physical States
The physical states at which some alkanes exist at room temperature
and atmospheric pressure are found in Table 20-5. Alkanes with the
lowest molecular mass, those with one to four carbon atoms, are gases.
Natural gas is a fossil fuel composed primarily of alkanes containing one
to four carbon atoms. The existence of these alkanes as gases agrees
with the idea that very small molecules have weak London dispersion
forces between them and are not held together tightly. Larger alkanes
are liquids. Gasoline and kerosene consist mostly of liquid alkanes.
Stronger London dispersion forces hold these molecules close enough
together to form liquids. Alkanes with a very high molecular mass are
solids, corresponding to a greater increase in London dispersion forces.
Paraffin wax contains solid alkanes. It can be used in candles, as shown
in Figure 20-12.

Boiling Points
The boiling points of alkanes, also shown in Table 20-5, increase with
increasing molecular mass. As London dispersion forces increase, more
energy, or heat, is required to pull the molecules apart. This property is
used in the separation of petroleum, a major source of alkanes. Petro- FIGURE 20-12 Paraffin wax, used
leum is a complex mixture of different hydrocarbons that varies greatly in candles, contains solid alkanes.
in composition. The hydrocarbon molecules in petroleum contain from Molecules of paraffin wax contain
one to more than fifty carbon atoms. This range allows the separation of 26 to 30 carbon atoms.

CARBON AND HYDROCARBONS 643 643

Copyright © by Holt, Rinehart and Winston. All rights reserved.


SECTION 20-3
TABLE 20-6 Petroleum Fractions
TABLE STRATEGY Size range of Boiling-point
Fraction molecules range (°C)
Table 20-6 Ask students why the
hydrocarbons with only one, two, Gasoline C4–C12 up to 200
and three carbon atoms are not on Kerosene C10–C14 180–290
this table. (The smaller hydrocarbons Middle distillate, such as heating C12–C20 185–345
are volatile and are not in the gaso- oil, gas-turbine fuel, diesel
line fraction.) Once again, be sure
students recognize the relationship Wide-cut gas oil, such as C20–C36 345–540
between the number of carbon atoms lubricating oil, waxes
and the boiling point. The larger Asphalt above C36 residues
hydrocarbons have higher boiling
points because they experience
stronger intermolecular attractions.

petroleum into different portions with different boiling point ranges, as


Common FIGURE 20-13 (a) Fractional shown in Table 20-6. In fractional distillation, shown in Figure 20-13,
Misconception distillation takes place in petroleum components of a mixture are separated on the basis of boiling point, by
Many students might think that oil refinery towers. (b) This is a model condensation of vapor in a fractionating column. During its fractional
was formed from decayed dinosaurs. of a fractional distillation tower. distillation, petroleum is heated to about 370°C. Nearly all the compo-
Although some of the organic mate- Because fractions contain hydro- nents of the petroleum are vaporized at this temperature. As the vapors
rial that makes up existing oil carbons of different masses, they rise in the fractionating column, or tower, they are gradually cooled.
reserves may have come from condense and are drawn off at
dinosaur tissue, the vast majority has different levels.
come from plants and bacteria. Refinery
tower
Gas
CHAPTER CONNECTION

Crude oil is a solution of different


Gasoline
hydrocarbons. Have students review
Chapter 13 to determine which prop- Kerosene
erties of solutions are mentioned in
this section.
Gas, oil

Visual Strategy Lubricating


FIGURE 20-13 Separate students oil stocks
into groups of three or four. Have
each group of students use the table Pipe still
on this page to determine the tem-
perature at different heights of the
Crude oil
distillation column shown in this vapor
figure. Have them brainstorm about
how a series of distillation columns (a)
can help produce a very pure product.

Residues
Crude oil
(b)

644 644 CHAPTER 20

Copyright © by Holt, Rinehart and Winston. All rights reserved.


SECTION 20-3
Alkanes with higher boiling points have higher condensation tempera-
tures and condense for collection lower in the tower. For example,
lubricating oils, which have higher condensation temperatures than SECTION REVIEW
gasoline has, are collected lower in the fractionating tower. 1. They contain only single bonds.
2. CH3
Combustion
Alkanes are less reactive than other hydrocarbons because of the sta- CH3ICICH3
bility of their single covalent bonds. One reaction alkanes do undergo is CH3
combustion. Because alkanes make up a large proportion of gaseous
and liquid fossil fuels, combustion is their most important reaction. CH3ICH2ICH2ICH2ICH3
Complete combustion of hydrocarbons produces energy, CO2, and CH3ICHICH2ICH3
H2O. The reaction for the combustion of methane produces 890 kJ/mol.
CH3
CH4 + 2O2 → CO2 + 2H2O 3. 2,2-dimethylpropane, pentane,
2-methylbutane
One concern about the combustion of fossil fuels is their possible 4. Answers will vary, depending on
contribution to the greenhouse effect. CO2 is one of the atmospheric which alkane is chosen. One example:
molecules that absorbs infrared radiation. Increased levels of CO2 alkanes with long carbon-atom
through the combustion of fossil fuels may increase the amount of chains have high boiling points and
infrared energy absorbed by the atmosphere to a level that can increase are used for structural purposes, such
the average temperature of Earth. as paving roads.
Engines can be powered by gasoline combustion. When fuel ignites 5. CH3ICH2 CH3
spontaneously before it is reached by the flame front, there is a decrease
CH3ICHIICHICH2ICH3
in the amount of power gained, and engine knocking results. Straight- heptane
chain hydrocarbons are more likely to ignite spontaneously than 3,4-dimethylhexane
branched-chain hydrocarbons. This tendency is the basis for the octane CH3 CH2ICH2ICH3
rating scale. The octane rating of a fuel is a measure of its burning effi-
ciency and its antiknock properties. The octane rating scale is based on
mixtures of 2,2,4-trimethylpentane, a highly branched alkane, and hep- 1-methyl-3-propylcyclopentane
tane, a straight-chain alkane. The term octane comes from the common
name of 2,2,4-trimethylpentane, isooctane. Pure 2,2,4-trimethylpentane 2,2,4-trimethylpentane
is very resistant to knocking and is assigned an octane number of 100.
FIGURE 20-14 The octane rating
Pure heptane has an octane number of 0 and burns with a lot of knock- scale is based on a rating of 100 for
ing. Increasing the percentage of branched-chain alkanes in gasoline is 2,2,4-trimethylpentane and 0 for
one way to increase octane rating. The octane rating on gasoline pumps heptane. Compare their molecular
is shown in Figure 20-14. shapes.

SECTION REVIEW
1. What is the basic structural characteristic of 4. Relate the properties of some alkanes to their
alkanes? uses.
2. Draw all of the condensed structural formulas 5. Draw the condensed structural formulas
that can represent C5H12. of 3,4-dimethylhexane and 1-methyl-3-
3. Give the systematic name for each of the com- propylcyclopentane.
pounds whose formulas appear in item 2.

CARBON AND HYDROCARBONS 645 645

Copyright © by Holt, Rinehart and Winston. All rights reserved.


SECTION 20-3

R E S E A R C H N O T E S
RESEARCH NOTES
Class Discussion
Prompt students through the reading
by asking the following questions:
1. What properties of diamond are Synthetic Diamonds
useful to industry? (hardness, heat
conductivity)
D iamonds made to order? Another method of coating crystalline molecular structure of
2. What are two methods used to
Almost. A thin coating of dia- with diamond, invented by metal- graphite, the spaces between car-
make artificial diamonds? (Chemical
mond film may not be pretty to lurgist Pravin Mistry, uses lasers to bon atoms are relatively far apart.
Vapor Deposition and laser formation
of plasma) look at, but it does offer many scan the object to be coated. The The process must compress the
useful properties to industry. A energy of the lasers breaks down spaces to form a compact octago-
3. How are the two methods for
number of methods are being CO2 (supplied by a gas delivery nal diamond crystal.
producing artificial diamonds similar?
developed to produce diamond system) into carbon and oxygen Diamond is one of the hardest
(Both methods separate carbon
atoms from carbon compounds and coatings cheaply and efficiently. If materials known to man, so dia-
recombine the carbon atoms alone.) successful, the processes will mond coatings would be particu-
affect the way tools, containers, larly useful for making machine
computer chips, and a host of tools, work surfaces, and other
other items are manufactured. applications where a durable pro-
James Adair is an associate pro- tective covering is needed.
fessor of material science and Diamond also has the highest
engineering at the University of thermoconductivity of any materi-
Florida. “Natural diamonds are al, which means that it transports
made at very, very high pressures heat very effectively. You wouldn’t
and heat,” Adair says. “Basically, want to drink from a diamond cof-
it’s a naturally occurring process fee cup because the cup would
that literally took millennia to warm up rapidly and you’d burn
form the diamond. We make dia- your lips. But diamond’s ability
monds in a couple of minutes.” to conduct heat makes it very
The process involves sticking very useful as a coating on silicon com-
This picture, taken with an electron
fine diamond particles on all kinds microscope, shows synthetic diamond puter chips.
of different surfaces. Chemical formed by Chemical Vapor Deposition. “For microelectronics,” says
Vapor Deposition is then used to Adair, “dealing with the heat
grow more diamond from these atoms and vaporizes the surface of generated by the circuit is one of
diamond “seeds.” the object, forming a superheated the biggest problems. If the heat
In Chemical Vapor Deposition, plasma. The plasma serves as an builds up too much within a sili-
the objects to be coated—in this environment for bonding the car- con circuit, it can literally melt the
case, the diamond seeds—are bon atoms into a coating of crys- silicon. And it’s not going to act as
placed inside a chamber filled with talline diamond. a very good computer brain for
methane and other gases. The One of the biggest challenges in you. Diamond can pull that heat
gases are subjected to microwave making synthetic diamond coatings out of the silicon chip, so the cir-
radiation, which breaks them is making sure that the carbon cuit can run a little bit cooler.”
down into hydrogen, carbon, and crystallizes correctly to form dia- If a computer chip is prevented
its mixtures (carbon-hydrogen mond and not graphite. Graphite is from getting too hot, it can per-
radicals). Diamond crystals grow useful for making lubricants and form faster. And a faster chip can
when these carbon atoms coat the pencil leads, but it is not as strong lead to a new breed of computers
diamond-seed crystals. and durable as diamond. In the with enhanced capabilities.

646 646 CHAPTER 20

Copyright © by Holt, Rinehart and Winston. All rights reserved.


SECTION 20-4

SECTION 20-4
Unsaturated Lesson Starter
During an earlier lesson, students
examined the homologous series of

Hydrocarbons OBJECTIVES alkanes—molecules in which carbon


atoms are saturated with hydrogen
atoms. Alkanes were described as
Distinguish between the hydrocarbons with the formulas that
structures of alkenes, alkynes, fit the relationship CnH2n+2. Alkenes
and aromatic hydrocarbons. are hydrocarbons that contain double
bonds. Have students think of dot
H ydrocarbons that do not contain the maximum amount of hydrogen Be able to name and write
structures that would accommodate
are referred to as unsaturated. Unsaturated hydrocarbons are hydro- the formula C3H6.
structural formulas for
carbons in which not all carbon atoms have four single covalent bonds. unsaturated hydrocarbons.
CHAPTER CONNECTION
Explain how structures of
Double bonds are stronger than
unsaturated hydrocarbons
Alkenes relate to their properties and
single bonds, but not twice as strong.
The initial single bond is much
how those properties affect
Alkenes are hydrocarbons that contain double covalent bonds. Some stronger and therefore has a higher
the uses of specific hydro-
examples of alkenes are given in Table 20-7. Notice that because alkenes bond energy than each additional
carbons.
have a double bond, the simplest alkene, ethene, has two carbon atoms. bond. Confirm this by reviewing bond
energies in Chapter 17.
Carbon atoms linked by double bonds cannot bind as many atoms as
those that are linked by only single bonds. An alkene with one double
bond has two fewer hydrogen atoms than the corresponding alkane. Reading Skill-Builder
COMPARING AND
H H H H
CONTRASTING Have students
HICICICIH HICJCICIH complete another section of their
H H H H H H table for unsaturated hydrocar-
NSTA bons, including basic structural
C3H8 C3H6
TOPIC: Alkenes characteristics, rules for naming,
Thus, the general formula for noncyclic alkenes with one double bond GO TO: www.scilinks.org properties, uses, formulas for, and
sci LINKS CODE: HC2205 examples of alkenes, alkynes, and
is CnH2n .
aromatic hydrocarbons. Then have
them use their tables to compare
and contrast saturated and unsatu-
TABLE 20-7 Structures of Alkenes rated hydrocarbons.
ethene propene trans-2-butene cis-2-butene
H H H H H CH3 H H
Structural CJC CJC CJC CJC
formula CH3
H H H CH3 H CH3 CH3

Ball-and-stick
model

CARBON AND HYDROCARBONS 647 647

Copyright © by Holt, Rinehart and Winston. All rights reserved.


SECTION 20-4
Because alkenes have a double bond, they can have geometric iso-

✔Teaching Tip
mers, as shown in the examples below.
H H H CH3
• Many students will have trouble CJC CJC
determining whether an alkene CH3 CH3 CH3 H
has geometric isomers. Have them
draw and examine the structural cis-2-butene trans-2-butene
formulas of each molecule in geo-
metric isomer form. Systematic Names of Alkenes
• An alkene is numbered so that The rules for naming a simple alkene are similar to those for naming an
the double bonds are closest to alkane. The parent hydrocarbon is the longest continuous chain of car-
carbon atoms with the lowest bon atoms that contains the double bond. If there is only one double
numbers. Have students compare bond, the suffix -ene is added to the carbon-chain prefix. Here, the
the structures of 2-pentene and longest chain that contains the double bond has five carbon atoms and
3-pentene. (They are both the one double bond, so the parent hydrocarbon is pentene.
same molecule.) Which is the cor-
rect name? (2-pentene; the name CH2ICH3 CH2ICH3
that has the double bond closest NOT
to the lowest-numbered carbon)
CH2JCICH2ICH2ICH3 CH2JCICH2ICH2ICH3
pentene hexane
The carbon atoms in the chain are numbered so that the first carbon
atom in the double bond has the lowest number. The number indicating
the position of the double bond is placed before the name of the hydro-
carbon chain and separated by a hyphen.

CH2ICH3
1 2 3 4 5
CH2JCICH2ICH2ICH3
1-pentene

The position number and name of the alkyl group are placed in front of
the double-bond position number. This alkyl group has two carbon atoms,
an ethyl group. It is on the second carbon atom of the parent hydrocarbon.
2-ethyl-1-pentene
The molecule is 2-ethyl-1-pentene.
If there is more than one double bond, the suffix is modified to indi-
cate the number of double bonds: 2 = -adiene, 3 = -atriene, and so on.

CH2JCHICH2ICHJCH2
1,4-pentadiene

If numbering from both ends gives equivalent positions for the double
bonds in an alkene with two double bonds, then the chain is numbered
from the end nearest the first alkyl group.

CH3
CH2JCICHJCH2

2-methyl-1,3-butadiene

648 648 CHAPTER 20

Copyright © by Holt, Rinehart and Winston. All rights reserved.


SECTION 20-4
The procedure for naming alkenes can be summarized as follows.

Alkene Nomenclature ADDITIONAL


SAMPLE
Use the rules for alkane nomenclature on page 639, with the
PROBLEM
following exceptions.
1. Name the parent hydrocarbon. Locate the longest continuous Additional Sample Problem is found
chain that contains the double bond(s). If there is only one double on page 661A.
bond, add the suffix -ene to the prefix corresponding to the num-
ber of carbon atoms in this chain. If there is more than one dou-
ble bond, modify the suffix to indicate the number of double
bonds. For example, 2 = -adiene, 3 = -atriene, and so on.
2. Add the names of the alkyl groups.
3. Number the carbon atoms in the parent hydrocarbon. Number the
carbon atoms in the chain so that the first carbon atom in the dou-
ble bond nearest the end of the chain has the lowest number. If
numbering from both ends gives equivalent positions for two dou-
ble bonds, then number from the end nearest the first alkyl group.
4. Insert position numbers. Place double-bond position numbers
immediately before the name of the parent hydrocarbon alkene.
Place alkyl group position numbers immediately before the name
of the corresponding alkyl group.
5. Punctuate the name.

SAMPLE PROBLEM 20- 3


Name the following alkene.
CH3
CH3ICHICJCH2
CH2ICH3

SOLUTION 1. Identify and name the parent hydrocarbon.


CH3
CH3ICHICJCH2
CH2ICH3
The parent hydrocarbon has four carbon atoms and one double bond, so it is named
butene.
2. Identify and name the alkyl groups.
CH3
CH3ICHICJCH2
CH3ICH3
The alkyl groups are ethyl and methyl.
Place their names in front of the name of the parent hydrocarbon in alphabetical order.
ethyl methyl butene

CARBON AND HYDROCARBONS 649 649

Copyright © by Holt, Rinehart and Winston. All rights reserved.


SECTION 20-4
3. Number the carbon chain to give the double bond the lowest position.

✔Teaching Tip 4
CH3
3 2 1
Ask students to interchange the CH3ICHICJCH2
methyl and ethyl groups in 2-ethyl- CH2ICH3
3-methyl-1-butene and then name
the resulting alkene. (The longest 4. Place the position number of the double bond in front of butene. Place the position numbers
chain contains five carbon atoms and of the alkyl groups in front of each alkyl group. Separate the numbers from the name with
the name is 2,3-dimethyl-1-pentene.) hyphens.
The first carbon in the double bond is in position 1.
Common The ethyl group is on carbon 2.
Misconception The methyl group is on carbon 3.
People often read the names of
chemicals in their food and cosmetics 2-ethyl-3-methyl-1-butene
and wince because they are intimi- The full name is 2-ethyl-3-methyl-1-butene.
dated by the IUPAC names that are
sometimes used. Some are afraid to
buy products that list ingredients
using IUPAC names. They do not PRACTICE 1. Name the following alkene: Answer
understand that any organic com- 2-hexene
pound, no matter how “natural,” can CH3ICH2ICH2ICHJCHICH3
be expressed using an IUPAC name. 2. Draw the condensed structural formula for Answer
4-methyl-1,3-pentadiene. CH2JCHICHJCICH3
Visual Strategy CH3
FIGURE 20-15 The IUPAC name of
α-farnesene is 3,7,11-trimethyl- 3. Name the following alkenes: Answer
1,3,6,10-dodecatetraene. a. CH3 a. 2-methyl-2-butene
CH3ICHJCHICH3 b. 2-methyl-3-hexene

b. CH3
CH3ICHICHJCHICH2ICH3

FIGURE 20-15 α-farnesene is Properties and Uses of Alkenes


a solid alkene found in the natural Alkenes are nonpolar and show trends in properties similar to those of
wax covering of apples. alkanes in boiling points and physical states. For example, α-farnesene
Can you determine has 15 carbon atoms and 4 double bonds, as shown in Figure 20-15.
the IUPAC This large alkene is a solid at room temperature and atmospheric
name for this
pressure. It is found in the natural wax covering of apples.
large alkene?
Ethene, the smallest alkene, is a gas.

CH3 CH CH2 CH CH CH
C CH2 C CH2 C CH2
CH3 CH3 CH3
α-farnesene

650 650 CHAPTER 20

Copyright © by Holt, Rinehart and Winston. All rights reserved.


SECTION 20-4
Ethene is the hydrocarbon commercially produced in the greatest
quantity in the United States. It is used in the synthesis of many plas-
tics and commercially important alcohols. Ethene is also an important
Application
The smallest alkene, ethene, is
plant hormone. Induction of flowering and fruit ripening, as shown in
responsible for stimulating fruit to
Figure 20-16, are effects of ethene hormone action that can be manipu- ripen. A chain reaction occurs as
lated by commercial growers. ripening fruit produces more ethene.
Tomatoes and avocados will ripen
more quickly in a bag because the
ethene is retained. One bad apple
Alkynes may indeed spoil the bunch because
of the ethene it gives off. Fruit grow-
Hydrocarbons with triple covalent bonds are alkynes. Like the double H H ers take advantage of this by picking
bond of alkenes, the triple bond of alkynes requires that the simplest CJC fruit before it is ripe, then exposing it
alkyne has two carbon atoms. H H to ethene before it goes to the shelf.

HICKCIH
ethyne

The general formula for the alkynes is CnH2n–2. Alkynes have four
fewer hydrogen atoms than the corresponding alkanes and two fewer
than the corresponding alkenes.

H H H H
HICICIH CJC HICKCIH
H H H H
FIGURE 20-16 Ethene is a plant
C2H6 C2H4 C2H2 hormone that triggers fruit ripening.
Its small size allows it to travel
as a gas.
Systematic Naming of Alkynes
Alkyne nomenclature is almost the same as alkene nomenclature. The
only difference is that the -ene suffix of the corresponding alkene is
replaced with -yne. A complete list of rules follows.

Alkyne nomenclature
Use the rules for alkane nomenclature on page 639, with the
following exceptions.
1. Name the parent hydrocarbon. Locate the longest continuous
chain that contains the triple bond(s). If there is only one triple
bond, add the suffix -yne to the prefix corresponding to the num-
ber of carbon atoms in the chain. If there is more than one triple
bond, modify the suffix to indicate the number of triple bonds. For
example, 2 = -adiyne, 3 = -atriyne, and so on.
2. Add the names of the alkyl groups.
3. Number the carbon atoms in the parent hydrocarbon. Number
the carbon atoms in the chain so that the first carbon atom in the
triple bond nearest the end of the chain has the lowest number.
If numbering from both ends gives the same positions for two triple
bonds, then number from the end nearest the first alkyl group.

CARBON AND HYDROCARBONS 651 651

Copyright © by Holt, Rinehart and Winston. All rights reserved.


SECTION 20-4
FIGURE 20-17 Ethyne is the
fuel used in oxyacetylene torches.
Additional Example
Oxyacetylene torches can reach
Problems temperatures of over 3000°C.
1. Name each of the following:
a. CH3ICH2ICKCH
b. CH3
CH3ICKCICH2ICHICH3
c. CH3
CHKCICICH3
CH3
Ans. a. 1-butyne
b. 5-methyl-2-hexyne
c. 3,3-dimethyl-1-butyne
4. Insert position numbers. Place the position numbers of the triple
2. Draw structural formulas for each bonds immediately before the name of the parent hydrocarbon
of the following:
alkyne. Place alkyl group position numbers immediately before
a. 2-butyne the name of the corresponding alkyl group.
b. 3-methyl-1-pentyne 5. Punctuate the name.
Ans. a. CH3ICKCICH3 Two examples of correctly named alkynes are given below.
b. CH3 CHKCICHICH3
CHKCICHICH2ICH3 CH3ICH2ICH2ICKCH
CH3
1-pentyne 3-methyl-1-butyne

Properties and Uses of Alkynes


Alkynes are nonpolar and exhibit the same trends in boiling points and
physical state as other hydrocarbons. The smallest alkyne, ethyne, is a
gas. The combustion of ethyne when it is mixed with pure oxygen pro-
duces the intense heat of welding torches, as shown in Figure 20-17. The
common name of ethyne is acetylene, and these welding torches are
commonly called oxyacetylene torches.

Aromatic Hydrocarbons
Aromatic hydrocarbons are hydrocarbons with six-membered carbon
rings and delocalized electrons. Benzene is the primary aromatic hydro-
carbon. The molecular formula of benzene is C6H6. One possible struc-
tural formula is a six-carbon atom ring with three double bonds.
H
H H

H H
H

652 652 CHAPTER 20

Copyright © by Holt, Rinehart and Winston. All rights reserved.


SECTION 20-4
However, benzene does not behave chemically like an alkene. All of
the carbon–carbon bonds in the molecule are the same. Like graphite,
benzene contains resonance hybrid bonds. The structure of the benzene H H Did You Know?
Friedrich August Kekulé claimed that
ring allows the delocalized electrons to be spread over the ring. The
H H
his structure of benzene came from
entire molecule lies in the same plane, as shown in Figure 20-18. The fol-
a dream. In his dream, he saw six
lowing structural formulas are often used to show this spreading of elec- snakes holding on to each other
trons. In the condensed form, the hydrogen atom at each corner is FIGURE 20-18 Electron orbitals
head to tail. When he woke up,
understood. in benzene overlap to form continu-
Kekulé realized that the only way the
ous orbitals that allow the delocal-
ized electrons to spread uniformly
structure would work is to have the
H
over the entire ring. carbon atoms line up in the same
H H
manner as the snakes in his dream.
Whether he truly solved the structure
of benzene from a dream or invented
H H the story to advance his theory ahead
H of others is debated.
Aromatic hydrocarbons can be thought of as derivatives of benzene.
The simplest have one benzene ring, as shown in the following example.
CHAPTER CONNECTION

CH3 Each carbon atom in benzene is sp 2


hybridized. To draw an accurate
Lewis dot structure, one must show
two resonance structures, with each
methylbenzene showing alternating double bonds.
To find out more about hybridization
and Lewis dot structures, see
Systematic Names of Aromatic Hydrocarbons Chapter 6.
The simplest aromatic hydrocarbons are named as alkyl-substituted
benzenes. The names of the alkyl groups are added in front of the word
benzene according to the rules for other hydrocarbons. As with NSTA

cycloalkanes, the carbon atoms in the ring do not need to be numbered TOPIC: Aromatic compounds
if there is only one alkyl group. If there is more than one alkyl group, GO TO: www.scilinks.org
sci LINKS CODE: HC2206
the carbons are numbered in order to give all of the alkyl groups the
lowest possible numbers. Following are some examples.
CH2ICH2ICH3
CH3 CH3

propylbenzene 1,3-dimethylbenzene
The rules for naming simple aromatic hydrocarbons can be summarized
as follows.

Simple Aromatic Hydrocarbon Nomenclature


Use the rules for alkane nomenclature on page 639, with the
following exceptions.
1. Name the parent hydrocarbon. The parent hydrocarbon is the
benzene ring, benzene.
2. Add the names of the alkyl groups.

CARBON AND HYDROCARBONS 653 653

Copyright © by Holt, Rinehart and Winston. All rights reserved.


SECTION 20-4
3. Number the carbon atoms in the parent hydrocarbon. If there are
ADDITIONAL two or more alkyl groups attached to the benzene ring, number
the carbon atoms in the ring. Assign position number one to the
SAMPLE
alkyl group that comes first in alphabetical order. Then number in
PROBLEMS
the direction that gives the rest of the alkyl groups the lowest
20-4 Draw structures for each of the numbers possible.
following: 4. Insert position numbers.
a. 1,3-dimethylbenzene 5. Punctuate the name.
b. 1-ethyl-2-methylbenzene
c. butylbenzene
Ans. a. CH3 SAMPLE PROBLEM 20- 4
Draw the condensed structural formula for 1,2-dimethylbenzene.

SOLUTION 1. Identify the parent hydrocarbon in the name.


CH3
1,2-dimethylbenzene
b. CH2ICH3
2. Draw the benzene ring.
CH3
c.
CH2ICH2ICH2ICH3

3. Number the carbon atoms in the benzene ring.


20-4 Each of the following names
represents an incorrect name for an 1
6 2
aromatic hydrocarbon. Replace each
5 3
name with the correct IUPAC name. 4
a. 2-methylbenzene
b. 1-ethyl-5-methylbenzene
4. Identify any alkyl groups.
c. 1-methyl-2-butylbenzene
Ans. a. 1-methylbenzene 1,2-dimethylbenzene
b. 1-ethyl-3-methylbenzene
c. 1-butyl-2-methylbenzene There are only methyl groups in this molecule. The prefix di- is attached to the word
methyl, so there are two methyl groups.

5. Locate the position numbers for the methyl groups.


1,2-dimethylbenzene

6. Attach the methyl groups to the carbon atoms numbered 1 and 2.


CH3
1 CH3
6 2
5 3
4

654 654 CHAPTER 20

Copyright © by Holt, Rinehart and Winston. All rights reserved.


SECTION 20-4
7. The complete structural formula for 1,2-dimethylbenzene is as follows.
CH3 Application
CH3 Both crude oil and gasoline contain
benzene, which is toxic and can be
absorbed through the skin. This is
why people should always protect
themselves when exposed to either
crude oil or gasoline.
PRACTICE 1. Name the following compound: Answer
ethylbenzene
CH2ICH3

2. Draw the condensed structural formula Answer


for 1-ethyl-4-methylbenzene.

CH3ICH2 CH3

SECTION REVIEW
1. a. double bonds
Properties and Uses of Aromatic Hydrocarbons b. triple bonds
Benzene rings are chemically very stable, a property that can be c. benzene rings
explained by the concept of delocalized electrons. Therefore, aromatic
2. CH2JCHICH2ICH3
hydrocarbons are less reactive than alkenes and alkynes are. In the past,
benzene was used as a nonpolar solvent because of this stability.
CH3ICHJCHICH3
However, benzene is both a poison and a carcinogen. Like other hydro-
carbons, benzene is nonpolar and has limited solubility in water. It CH3ICJCH2
appears that oxidation of the benzene ring, in an attempt to solubilize it
for elimination from the body, produces toxic molecules. This has led to CH3
the replacement of benzene as a solvent with methylbenzene, which is Other answers are also possible,
less toxic. Another aromatic hydrocarbon, 3,4-benzpyrene, is found in such as:
ICH3
coal tar, tar from cigarette smoke, and soot in heavily polluted urban
areas. Studies have shown this compound can cause cancer. 3. 1-butene, 2-butene, 2-methyl-1-
propene; some students may also
think of cyclobutane and methyl-
cyclopropane.
SECTION REVIEW 4. Answers will vary. Possible
responses include the following:
1. List the basic structural features that characterize 4. Give examples of a property or use of three the delocalized electrons of benzene
each of the following: unsaturated hydrocarbons. make it very stable, large alkenes
a. alkenes are solid at room temperature, and
5. Draw the condensed structural formula for each ethyne is used as fuel for welding
b. alkynes of the following:
c. aromatic hydrocarbons torches.
a. 1,3-butadiene
2. Draw three condensed structural formulas that b. 2-pentyne 5. a. CH2JCHICHJCH2
can represent C4H8. c. 1,2-diethylbenzene b. CH3ICKCICH2ICH3
3. Give the systematic name for each compound in c. CH2ICH3
your answer to item 2.

CH2ICH3

CARBON AND HYDROCARBONS 655 655

Copyright © by Holt, Rinehart and Winston. All rights reserved.


CHAPTER 20 REVIEW
CHAPTER 20 REVIEW
REVIEW ANSWERS
CHAPTER SUMMARY
1. They are oriented toward the four
corners of a regular tetrahedron.
20-1 • Carbon is important because all living matter • Carbon occurs in several solid allotropic forms,
2. diamond, graphite, fullerenes;
in diamond, each carbon atom is contains carbon. such as diamond, graphite, and fullerenes, all of
tetrahedrally oriented to its four • Hybridized orbitals allow carbon atoms to form which have different structures and properties.
nearest neighbors; in graphite, single, double, and triple covalent bonds.
carbon atoms are arranged in Vocabulary
thin hexagonal plates; in delocalized electrons (627) diamond (626) fullerenes (626) graphite (626)
fullerenes, carbon atoms
compose near-spherical cages. 20-2 • All organic compounds contain carbon, but not • Isomers are compounds with the same molecular
3. hardness and high melting point all carbon-containing compounds are classified formula but different structures. Structural for-
4. Graphite has delocalized elec- as organic. mulas are needed to show the bonding order
trons, while diamond does not. • The number of possible organic compounds is and arrangement of atoms in an organic mol-
The delocalized electrons move in virtually unlimited because of the bonding prop- ecule to distinguish between isomers.
an electric field, allowing graphite erties of carbon. The unique catenation ability • Structural isomers are isomers in which the
to conduct electricity. of carbon allows it to link together to form long atoms are bonded together in different orders.
5. Graphite’s structure consists of chains and rings. The ability of carbon to bind Geometric isomers are isomers in which the
layers that can slide across one other elements and to allow different arrange- order of atom bonding is the same but the atoms
another. ments of atoms adds to the diversity of carbon are arranged differently in space.
6. spherical, soccer-ball shape compounds.
Vocabulary
7. a. Catenation is the covalent
binding of an element to itself catenation (630) hydrocarbons (630) organic compounds (629) structural isomers (631)
to form chains or rings. geometric isomers (632) isomers (630) structural formula (630)
b. Catenation contributes to
organic diversity by allowing 20-3 • In saturated hydrocarbons, each carbon atom bonds, these compounds are not very reactive.
carbon atoms to bond together has four single covalent bonds. Alkanes are satu- One important reaction they do undergo is
in many possible shapes. rated hydrocarbons. combustion.
8. Hydrocarbons are molecules • Organic compounds are named according to a • Trends in physical states, boiling points, and
composed of hydrogen and systematic method developed by IUPAC. combustion properties correspond to trends
carbon. They are important • Alkanes contain only single bonds. Because in alkane size and amount of branching.
because they are the backbones alkanes consist of saturated single covalent
of other organic compounds. Vocabulary
9. a. the exact number and types alkanes (634) fractional distillation (644) natural gas (643) petroleum (643)
of atoms in a molecule and the alkyl groups (637) homologous series (634) octane rating (645) saturated hydrocarbons (634)
order in which they are bonded cycloalkanes (635)
together
b. to show the structure of a 20-4 • Carbon atoms in unsaturated hydrocarbons do alkene, ethene, is an important industrial and
molecule and distinguish one not all have four single covalent bonds. Alkenes, agricultural chemical.
isomer from another alkynes, and aromatic hydrocarbons are unsatu- • Alkynes contain carbon-carbon triple bonds.
10. No. They have different formulas. rated hydrocarbons. • Benzene and derivatives of benzene are aromatic
11. No. They do not have rigid • Alkenes contain carbon-carbon double bonds hydrocarbons. The concept of delocalized electrons
structures. and can have geometric isomers. The smallest helps explain the stability of the benzene ring.
12. a. A saturated hydrocarbon has Vocabulary
as many hydrogen atoms as alkenes (647) aromatic hydrocarbons (652) benzene (652) unsaturated hydrocarbons (647)
possible bonded with single alkynes (651)

656 656 CHAPTER 20

Copyright © by Holt, Rinehart and Winston. All rights reserved.


CHAPTER 20 REVIEW CHAPTER 20 REVIEW

REVIEWING CONCEPTS b. CH3ICHJCH2 covalent bonds to every carbon


atom. An unsaturated hydrocar-
1. What is the orientation of the four covalent bonds c. CH3
bon does not have as many
and the sp3 orbitals of a carbon atom? (20-1) CHKCICHICH2ICH3 hydrogen atoms because not all
2. Name and describe the structures of three d. CH3ICHICH2ICH2ICH2ICH2ICH3 carbon atoms have four single
allotropic forms of carbon. (20-1) covalent bonds.
CH3 b. whether or not a solution con-
3. What properties of diamond determine most of (20-3 and 20-4) tains the maximum amount of
its industrial uses? (20-1) dissolved solute possible
14. Give the general formula for the members of
4. Why does graphite conduct electricity while the following: c. Alkanes are saturated. Alkenes,
diamond does not? (20-1) alkynes, and aromatic hydrocar-
a. alkane series
bons are unsaturated.
5. Explain why the structure of graphite makes it b. alkene series
useful as a lubricant. (20-1) c. alkyne series (20-3 and 20-4) 13. a. aromatic hydrocarbon
b. alkene
6. Describe the structure of buckminster- 15. Give the molecular formula for each type of c. alkyne
fullerene. (20-1) hydrocarbon if it contains seven carbon atoms. d. alkane
7. a. What is catenation? a. an alkane
14. a. CnH2n+2
b. How does catenation contribute to the b. an alkene b. CnH2n
diversity of organic compounds? (20-2) c. an alkyne (20-3 and 20-4) c. CnH2n-2
8. What are hydrocarbons, and what is their 16. a. What is a homologous series? 15. a. C7H16
importance? (20-2) b. By what method are straight-chain hydrocar- b. C7H14
bons named? c. C7H12
9. a. What information about a compound is c. Name the straight-chain alkane with the
provided by a structural formula? 16. a. a series in which adjacent
molecular formula C10H22. (20-3) members differ by a constant
b. How are structural formulas used in organic
17. What are cycloalkanes? (20-3) unit
chemistry? (20-2)
b. Add the suffix -ane to the pre-
10. Can molecules with the molecular formulas 18. a. What trend occurs in the boiling points of fix that corresponds to the num-
C4H10 and C4H10O be structural isomers of one alkanes? ber of carbon atoms in the chain.
another? Why or why not? (20-2) b. How would you explain this trend? c. decane
c. How is the trend in alkane boiling
11. Can molecules with only single bonds 17. saturated hydrocarbons that
points used in petroleum fractional contain ring structures
(and no rings) have geometric isomers? distillation? (20-3)
Why or why not? (20-2) 18. a. As the number of carbon
19. How does the structure of alkanes affect the atoms in alkanes increases,
12. a. What do the terms saturated and unsaturated octane rating of gasoline? (20-3) so does their boiling point.
mean when applied to hydrocarbons?
20. Write a balanced equation for the complete b. The increased boiling point
b. What other meanings do these terms have in in larger alkanes is due to
chemistry? combustion of each of the following:
greater intermolecular attraction
c. Classify alkenes, alkanes, alkynes, and a. methane
(dispersion forces).
aromatic hydrocarbons as either saturated b. ethyne (20-3 and 20-4)
c. Petroleum products are dis-
or unsaturated. (20-3 and 20-4) 21. Which types of isomers are possible for alkanes tilled in various fractions divided
13. Classify each of the following as an alkane, (with no rings), alkenes, and alkynes? by similar boiling points. In a dis-
Why? (20-3 and 20-4) tillation tower, the products with
alkene, alkyne, or aromatic hydrocarbon.
lower boiling points condense at
a. CH2ICH3 22. Give examples of ethene’s commercial uses.
the top, where it is cooler. The
(20-4) larger fractions with higher boil-
23. a. Alkyne nomenclature is very similar to the ing points condense and are
nomenclature of what other group of removed near the bottom.
CH2ICH3 hydrocarbons?
b. How do these nomenclatures differ? (20-4)

CARBON AND HYDROCARBONS 657 657

Copyright © by Holt, Rinehart and Winston. All rights reserved.


CHAPTER 20 REVIEW CHAPTER 20 REVIEW

24. Give one use for ethyne. (20-4) CH3ICHICHICH2ICH3


19. A higher percentage of
branched-chain alkanes 25. a. What are delocalized electrons? CH3 CH3
increases the octane rating. b. What is their effect on the reactivity of
20. a. CH4 + 2O2 → 2H2O + CO2 aromatic hydrocarbons? (20-4)
Isomers
b. 2C2H2 + 5O2 → 2H2O + 4CO2 26. What is the name of the parent hydrocarbon of 30. Identify whether each pair represents the same
21. All can have structural isomers simple aromatic hydrocarbons? (20-4) molecule or structural isomers.
because of the bonding capability 27. Describe a possible cause of benzene
of carbon atoms. Alkanes (with toxicity. (20-4) a. CH3ICH2ICH2ICH3 CH3
no rings) cannot have geometric
isomers because they have no CH2ICH2ICH3
rigid structures. Alkenes can have PROBLEMS b. CH3
geometric isomers because they
have double bonds—rigid struc- Structural Formulas CH3ICHICH2ICH2
tures. Alkynes cannot have geo- 28. Draw the condensed structural formula for the CH2
metric isomers because they following:
cannot have two different groups CH3
attached to each carbon atom in H H H CH3
a triple bond. HICJC C CIH CH3ICHICH2ICHICH3
22. to ripen fruit, to induce flower- H HICIH H CH3
ing, and to make plastics and
H
commercially important alcohols c. O O
23. a. alkenes 29. Identify each of the following pairs of formulas CH3ICH2ICIOH CH3IOICH2ICH
b. The names of alkenes end as representing the same or different molecules:
with -ene. The names of alkynes 31. Draw structural formulas for the five isomers of
a. C5H12 AND H H H H C6H14.
end with -yne.
HICIC C CIH
24. combustion in welding torches 32. Draw the geometric isomers of the following
H H HICIH H molecule. Label each isomer as cis or trans.
25. a. electrons that are shared
between more than two atoms H
CH3ICHJCHICH2ICH3
b. Delocalized electrons b. CH3ICH2ICH3
contribute to the stability 33. a. Which of the following can have geometric
of aromatic hydrocarbons. AND isomers?
26. benzene CH3
H H H H
27. When the human body oxidizes CH3ICHJCHICl CH3ICHJCICH3
HICICICICIH
benzene in an attempt to solu- CH3ICH2ICHJCHICH2ICH3
bilize it for elimination, toxic H H H H
products are formed. b. Draw the geometric isomers for those that
c. C6H10 AND CH3ICHJCICH3 can have geometric isomers.
28. See page 661A for structure.
c. Label each geometric isomer as cis or trans.
29. a. same c. different CH3
b. different d. same d. H H H H H Alkane Nomenclature
30. a. same HIC C C CICIH 34. Name the following molecules. (Hint: See
b. structural isomers
c. structural isomers H HICIH HICIH H H Sample Problem 20-1.)

31. See page 661A for structure. H H a. CH3ICH2ICH2ICH2ICH2ICH2ICH3


32. See page 661A for structure. AND b.
33. See page 661A for structure.

658 658 CHAPTER 20

Copyright © by Holt, Rinehart and Winston. All rights reserved.


CHAPTER 20 REVIEW CHAPTER 20 REVIEW

c. CH3 c. CH3 34. a. heptane


CH3ICICH2ICHICHICH3 CH2JCHICICH2ICH3 b. cycloheptane
c. 2,2,4-trimethylhexane
CH3 CH3 CH2
d. 5-butyl-2,2-dimethylnonane
CH3
d. CH3 CH2ICH2ICH2ICH3 35. See page 661A for structure.
CH3ICICH2ICH2ICHICH2ICH2ICH2ICH3 d. CHJCICH2ICH2ICHJCH2 36. See page 661A for structure.
CH3 CH3 37. a. butane
35. Give the complete, uncondensed, structural 39. Draw the condensed structural formula for b. correct
formula for each of the following alkanes. c. 3-methylhexane
each of the following alkenes:
d. correct
(Hint: See Sample Problem 20-2.) a. 2-methyl-2-hexene
a. decane b. 3-ethyl-2,2-dimethyl-3-heptene 38. a. 1-pentene
b. 3,3-dimethylpentane b. 2-methyl-2-butene
40. Draw structural formulas for geometric isomers c. 3-ethyl-3-methyl-1-pentene
36. Give the condensed structural formula for of each of the following: d. 2-methyl-1,5-hexadiene
each of the following alkanes: a. CH3ICH2ICH2ICHJCHICH3
39. See page 661B for structure.
a. 1,1-dimethylcyclopropane b. 3-methyl-2-pentene
b. 2,2,4,4-tetramethylpentane 40. See page 661B for structure.

37. For each of the following, determine whether 41. a. propyne


Alkyne Nomenclature b. 4-methyl-2-pentyne
the alkane is named correctly. If it is not, give 41. Name the following alkynes: c. 2,5-dimethyl-3-hexyne
the correct name. a. CHKCICH3 d. 1,6-heptadiyne
a. CH3ICH2ICH2 42. See page 661B for structure.
b. CH3ICKCICHICH3
CH3 43. a. benzene
CH3
1-methylpropane b. 1,2,3,5-tetramethylbenzene
c. CH3ICHICKCICHICH3
b. CH3ICH2ICH2ICH2ICH2ICH2ICH2 44. See page 661B for structure.
CH2 CH3 CH3 45. 0.0570 cm3
CH3 d. CHKCICH2ICH2ICH2ICKCH 46. a. 1.85 mol
nonane b. 1.11 × 1024 molecules
42. Draw the condensed structural formula for each
of the following alkynes: 47. C = 85.6%; H = 14.4%
c. CH3
a. 1-decyne 48. 45.0 L
CH3ICH2ICH2ICHICH2ICH3
b. 6,6-dimethyl-3-heptyne 49. 31.4 kg
4-methylhexane
50. a-b. See page 661B for structure.
d. CH3 Aromatic Hydrocarbon Nomenclature
c. from the structural formula:
43. Name the following aromatic hydrocarbons. C9H20; from the general formula:
CH3ICH2ICHICH2ICHICH3
(Hint: See Sample Problem 20-4.) CnH2n +2 = C9H2 × 9+2 = C9H20 ;
CH2ICH3
a. they are the same.
4-ethyl-2-methylhexane
51. See page 661B for structure.
Alkene Nomenclature 52. See page 661B for structure.
38. Name the following alkenes. (Hint: See Sample 53. See page 661B for structure.
Problem 20-3.) b. CH3
54. a. no geometric isomers
CH3 b. See page 661B for structure.
a. CH2JCHICH2ICH2ICH3
c. no geometric isomers
b. CH3 H d. no geometric isomers
CJC CH3 CH3
CH3 CH3

CARBON AND HYDROCARBONS 659 659

Copyright © by Holt, Rinehart and Winston. All rights reserved.


CHAPTER 20 REVIEW CHAPTER 20 REVIEW

44. Draw the condensed structural formula for each 53. Draw the three structural isomers for an alkyne
55. a. same compound
b. different compounds, not
of the following molecules: containing five carbon atoms and one triple
isomers a. 1,3,5-trimethylbenzene bond. Name the molecules you draw.
c. same compound b. 1,3-dimethylbenzene 54. Which of the following molecules have
d. isomers geometric isomers? Draw all possible geometric
56. Ionic bonds are usually stronger Calculations with Carbon Compounds isomers. Label the molecules you draw as either
than covalent bonds; more en- 45. The jewelers’ mass unit for diamond is the carat. cis or trans.
ergy is needed to break ionic By definition, 1 carat equals exactly 200 mg. a. butane
bonds. What is the volume of a 1.00 carat diamond? b. 2-pentene
57. Almost all carbon compounds The density of diamond is 3.51 g/cm3. c. 2-hexyne
contain hydrogen. In addition, 46. For 100.0 g of butadiene, C4H6, calculate the d. 2-methyl-1-butene
hydrogen is contained in acids, following: 55. Identify the following pairs as the same
hydroxides, ammonium
a. number of moles b. number of molecules compound, isomers, or different compounds
compounds, and many other
47. An alkene has the molecular formula C12H24. that are not isomers:
inorganic compounds.
58. The percentage of hydrogen Determine its percent composition. a. H H H H H
decreases as the number of 48. Assuming that the volumes of carbon dioxide HIC C C CICIH
carbon atoms in an alkane and of propane are measured under the same H HICIH H H H
increases. experimental conditions, what volume of carbon H
59. a. ethylene and propylene dioxide is produced by the complete combus-
tion of 15.0 L of propane? AND
b. CH2JCH2 CH2JCH–CH3
ethylene propylene 49. Assume a gasoline is isooctane, which has a CH3ICHICH2ICH2ICH3
c. They are both alkenes. density of 0.692 g/mL. What is the mass in CH3
60. a. the amount of oxygen present kilograms of 12.0 gal of the gasoline
(1 gal = 3.78 L)? b. C4H8
b. CO is a deadly poison that
binds more readily than O2 with AND
heme in the blood. Cells die from
MIXED REVIEW H H H H
a lack of O2.
HICICICICIH
61. a. Silicon bonds to oxygen rather 50. a. Draw the complete, uncondensed structural
than to other silicon atoms. formula for 4-methyloctane. H H H H
Carbon bonds directly to other b. Convert it into the condensed structural c. CH3
carbon atoms. formula.
b. SiO4, tetrahedral CH3 CH2
c. Determine the molecular formula for the
62. a. See page 661B for structure. CH3ICIIICH2
molecule from both the structure you drew
b. dimethyl mercury and the general molecular formula for alkanes. CH3
63. The actual list of compounds Compare the two. Are they the same? AND
published will vary year to year.
51. Draw and name two different condensed
64. Student lists will vary. Be sure CH3
structural formulas for molecules of each of
students are considering petro- the following types of hydrocarbons containing CH3ICICH2ICH2ICH3
leum products, such as clothes eight carbon atoms: CH3
and food, that may not be imme-
a. alkane c. alkyne
diately obvious. d. CH3ICJCHICH2ICH3
b. alkene d. aromatic hydrocarbon
65. Try not to model large molecules CH3
with this method; it becomes 52. Draw the condensed structural formulas for
very difficult for the models to 4,4-dimethyl-2-pentyne and 2,2-dimethyl-4- AND
keep their shape. propyloctane. CH3ICH2ICHICHJCH2
CH3

660 660 CHAPTER 20

Copyright © by Holt, Rinehart and Winston. All rights reserved.


CHAPTER 20 REVIEW CHAPTER 20 REVIEW

CRITICAL THINKING differ in composition from a long-chain 66. This activity will focus on the
carbon compound? different groups on both sides of
56. Inferring Conclusions Why are organic com- the double bond. The gumdrop
b. The simplest alkane is methane. Methyl
pounds with covalent bonds usually less stable models will easily show how
groups are found in all alkanes. What is a
when heated than inorganic compounds with there can be free rotation around
common subunit of a silicate? What is the
ionic bonds? a single bond. When two tooth-
geometry of that subunit?
57. Inferring Relationships The element that picks are used to make the dou-
62. Mercury in the environment poses a hazard to ble bond, they will prevent
appears in the greatest number of compounds is
living things. Review the section on mercury rotation.
hydrogen. The element found in the second
poisoning in the Elements Handbook.
greatest number of compounds is carbon. Why
a. Draw a structure formula for the organic
are there more hydrogen compounds than car-
mercury compound described in that sec-
bon compounds?
tion.
58. Relating Ideas As the number of carbon b. What is the IUPAC name for this compound?
atoms in an alkane molecule increases, does the
percentage of hydrogen increase, decrease, or
remain the same? RESEARCH & WRITING
63. Chemical and Engineering News publishes a list
once a year of the top 50 chemicals. Find out
HANDBOOK SEARCH which chemicals on the current year’s list are
59. The top 10 chemicals produced in the United hydrocarbons, and report your findings to the
States are listed in Table 7B of the Elements class.
Handbook. Review this material, and answer 64. Consult reference materials at the library, and
the following: read about products made from hydrocarbons.
a. Which of the top ten compounds are Keep a list of the number of petroleum-related
organic? products you use in a single day.
b. Write structural formulas for the compounds
you listed in item (a).
c. To what homologous series do each of ALTERNATIVE ASSESSMENT
these compounds belong?
65. Performance Models are often used to visual-
60. The reaction of methane with oxygen produces ize the three-dimensional shape of molecules.
two different oxides of carbon. Review this Using gumdrops as atoms and toothpicks to
material in the Elements Handbook, and bond them together, construct models of differ-
answer the following: ent hydrocarbons. Use large gumdrops for
a. What conditions determine whether the carbon and smaller gumdrops for hydrogen.
product of the methane reaction is CO2 Refer to Figures 20-1 and 20-2 for guidelines on
or CO? the three-dimensional shapes of hydrocarbons.
b. If a home heating system is fueled by natur-
66. Performance Using your gumdrop models,
al gas, what difference does it make if the
demonstrate why alkenes can have geometric
combustion produces CO2 or CO?
isomers, while alkanes cannot.
61. Silicon is similar to carbon in forming long-
chain compounds. Review the material on
silicon in the Elements Handbook and answer
the following.
a. How does a long-chain silicon compound

CARBON AND HYDROCARBONS 661 661

Copyright © by Holt, Rinehart and Winston. All rights reserved.


CONTINUATION OF ANSWERS

ADDITIONAL 20-2 CH ICH ICHICH ICH ICH 32. CH3 CH ICH3


3 2 2 2 3 O P 2
SAMPLE CH3 CJC cis
P O
PROBLEMS H H
CH3ICH2ICH2ICH2ICH2ICH3ICH3
Additional Sample Problem from page 639 (Other structures are also possible.) CH3 H
O P
CJC trans
20-1 Name the following simple 20-2 CH3 CH2ICH3 P O
branched-chain alkanes: H CH2ICH3
CH3ICIIICHICH2ICH3
a. CH3ICH2ICH2ICHICHICH3 33. a. CH3ICHJCHICl
CH3
CH3 CH3 CH3ICH2ICHJCHICH2ICH3
3-ethyl-2,2-dimethylpentane
b. CH3ICH2ICH2ICHICHICH2ICH2ICH3
Additional Sample Problem from page 649 b-c Cl CH
CH3 CH3 O P 3
CJC cis
c. CH2ICH2ICHICH3 20-3 Name the following alkenes: P O
a. CH3ICH2ICHICH2ICH2ICHJCH2 H H
CH3
CH3 Cl H
d. CH3ICH2ICHICHICH2 O
CJC
P
b. CH3ICHJCHICHJCHICH3 P O trans
CH3 CH3 CH2ICH2ICH2ICH3 H CH3
c. CH3ICHICHJCHICH2ICH2ICH3
e. CH3ICH2ICHICH2ICH2ICH2ICH3 CH3ICH2 CH ICH3
CH3 O P 2
CH3 CJC cis
P O
Ans. a. 5-methyl-1-heptene H H
Ans. a. 2,3-dimethylhexane
b. 2,4-hexadiene
b. 4,5-dimethyloctane CH3ICH2 H
c. 2-methyl-3-heptene
c. 2-methylbutane
O P
CJC trans
d. 3,4-dimethylnonane H
P O
CH2ICH3
e. 3-methylheptane
REVIEW ANSWERS
Answers from page 659
Answers from page 640
Answers from page 658 35. a. H H H H H H H H H H
20-2 a. 2,4-dimethylpentane
28. CH2JCHICHICH3 HICICICICICICICICICICIH
CH3ICHICH2ICHICH3
CH3 H H H H H H H H H H
CH3 CH3
31. CH3ICH2ICH2ICH2ICH2ICH3 b. H
b. 4-ethyl-3-methylheptane
CH3ICH2ICHICHICH2ICH2ICH3 HICIH
CH3ICH2ICHICH2ICH3
CH3 CH2ICH3 H H H H
CH3
HICICICICICIH
c. 2-methylpropane
CH3ICHICH2ICH2ICH3 H H H H
CH3ICHICH3 HICIH
CH3
CH3 H
CH3
CH3ICHICHICH3 36. a. CH3 CH3
Answers from page 641
20-2 CH3ICH2 CH2ICH3 CH3
CH3ICH2ICIIICICH2ICH3 CH3
b. CH3 CH3
CH3ICH2 CH3 CH3ICICH2ICH3
CH3ICIIICH2IIICICH3
3,3,4-triethyl-4-methylhexane CH3
CH3 CH3

661A

Copyright © by Holt, Rinehart and Winston. All rights reserved.


CONTINUATION OF ANSWERS
39. a. CH3 51. Answers will vary. 53. a. CHKCICH2ICH2ICH3
CH3ICJCHICH2ICH2ICH3 a. CH3
1-pentyne
CH3ICICH2ICH2ICH2ICH3
b. CH3 CH3ICKCICH2ICH3
CH3
CH3ICIIICJCHICH2ICH2ICH3 2-pentyne
2,2-dimethylhexane
CH3 CH2ICH3 CHKCICHICH3
CH3
40. a. CH3ICH2ICH2 CH CH3
O P 3 CH3ICH2ICH2ICHICH2ICH2ICH3
3-methyl-1-butyne
CJC
P O 4-methylheptane
H H 54. b. CH3 CH ICH3
b. CH3 O P 2
CJC cis
CH3ICH2ICH2 H P O
O P CH3ICH2ICHJCHICICH3 H H
CJC
H
P O
CH3 CH3 CH3 H
O P
2,2-dimethyl-3-hexene CJC trans
b. CH3 CH ICH3 P O
O P 2 CH3 CH3 CH3 H CH2ICH3
CJC
P O
H CH3 CH3ICHICJJCICH3
Answer from page 661
2,3,4-trimethyl-2-pentene H
H CH ICH3 62. a.
O P 2 c. CH3 CH3
CJC HICJCICIH
P
CH3
O
CH3 CH3ICKCICHICHICH3
H H H
4,5-dimethyl-2-hexyne
42. a. CHKCICH2ICH2ICH2ICH2
I

CHKCICH2ICHICH2ICH3
CH3ICH2ICH2ICH2
CH2ICH3
b. CH3
4-ethyl-1-hexyne
CH3ICH2ICKCICH2ICICH3
d. CH3
CH3
44. a. CH3
CH3
1,2-dimethylbenzene
CH2ICH3
CH3 CH3
b. CH3

ethylbenzene
52. CH3
CH3 CH3ICKCICICH3
50. a. H CH3
HICIH 4,4-dimethyl-2-pentyne
H H H H H H H CH3 CH2ICH2ICH3
HICICICICICICICICIH CH3ICICH2ICHICH2ICH2ICH2ICH3
H H H H H H H H CH3
b. CH3 2,2-dimethyl-4-propyloctane
CH3ICH2ICH2ICHICH2ICH2ICH2ICH3

661B

Copyright © by Holt, Rinehart and Winston. All rights reserved.

Anda mungkin juga menyukai